Решение уравнений онлайн с решением методом гаусса: Онлайн калькулятор. Решение систем линейных уравнений. Метод Гаусса.

Содержание

вычисление матрицы онлайн методом гаусса онлайн

Вы искали вычисление матрицы онлайн методом гаусса онлайн? На нашем сайте вы можете получить ответ на любой математический вопрос здесь. Подробное решение с описанием и пояснениями поможет вам разобраться даже с самой сложной задачей и вычислить систему уравнений онлайн, не исключение. Мы поможем вам подготовиться к домашним работам, контрольным, олимпиадам, а так же к поступлению в вуз. И какой бы пример, какой бы запрос по математике вы не ввели — у нас уже есть решение. Например, «вычисление матрицы онлайн методом гаусса онлайн».

Применение различных математических задач, калькуляторов, уравнений и функций широко распространено в нашей жизни. Они используются во многих расчетах, строительстве сооружений и даже спорте. Математику человек использовал еще в древности и с тех пор их применение только возрастает.

Однако сейчас наука не стоит на месте и мы можем наслаждаться плодами ее деятельности, такими, например, как онлайн-калькулятор, который может решить задачи, такие, как вычисление матрицы онлайн методом гаусса онлайн,вычислить систему уравнений онлайн,гаусс калькулятор,гаусс онлайн,гаусса матрица онлайн,гаусса метод решения систем линейных уравнений онлайн,гаусса онлайн,гаусса онлайн калькулятор,гаусса онлайн решение,гаусса решение онлайн,гауссом решение онлайн,жордана гаусса калькулятор,исследовать на совместность систему онлайн,исследовать систему и если она совместна найти решение онлайн,исследовать систему на совместность онлайн калькулятор,исследовать совместность и найти общее решение системы онлайн,исследовать совместность системы и найти общее решение онлайн,как решить матрицу методом гаусса онлайн,как решить матрицу онлайн методом гаусса,калькулятор гаусс,калькулятор гаусса,калькулятор гаусса жордана,калькулятор гаусса жордана гаусса онлайн калькулятор,калькулятор гаусса онлайн,калькулятор гаусса с подробным решением,калькулятор для матриц метод гаусса,калькулятор для метода гаусса,калькулятор для решения линейных уравнений,калькулятор для решения уравнений линейных,калькулятор для систем уравнений,калькулятор для системы уравнений онлайн,калькулятор жордана гаусса,калькулятор линейного уравнения,калькулятор линейное уравнение,калькулятор линейные уравнения,калькулятор линейных уравнений,калькулятор линейных уравнений онлайн,калькулятор матриц гаусс,калькулятор матриц гаусса,калькулятор матриц гаусса онлайн,калькулятор матриц метод гаусса,калькулятор матриц метод гаусса онлайн,калькулятор матриц метод гаусса с решением,калькулятор матриц методом гаусса,калькулятор матриц методом гаусса онлайн,калькулятор матриц методом гаусса онлайн калькулятор,калькулятор матриц методом гаусса с решением онлайн,калькулятор матриц методом жордана гаусса онлайн калькулятор,калькулятор матриц онлайн гаусса,калькулятор матриц онлайн метод гаусса,калькулятор матриц онлайн методом гаусса,калькулятор матриц онлайн с решением метод гаусса,калькулятор матриц онлайн с решением методом гаусса,калькулятор матриц онлайн с решением методом гаусса онлайн,калькулятор матриц по методу гаусса,калькулятор матриц решение методом гаусса,калькулятор матриц с решением метод гаусса,калькулятор матрица метод гаусса,калькулятор матрицы гаусса,калькулятор матрицы метод гаусса,калькулятор матрицы методом гаусса,калькулятор матрицы методом гаусса онлайн,калькулятор матрицы онлайн метод гаусса,калькулятор матрицы онлайн методом гаусса,калькулятор матрицы онлайн с решением метод гаусса,калькулятор матричный метод гаусса,калькулятор метод гаусса,калькулятор метод гаусса жордана,калькулятор метод гаусса онлайн с решением,калькулятор метод гаусса решения систем линейных уравнений онлайн,калькулятор метод гаусса с подробным решением,калькулятор метод гаусса с решением,калькулятор метод жордана гаусса,калькулятор метода гаусса,калькулятор методом гаусса,калькулятор методом гаусса онлайн,калькулятор онлайн для системы уравнений,калькулятор онлайн линейное уравнение,калькулятор онлайн линейные уравнения,калькулятор онлайн линейных уравнений,калькулятор онлайн матриц гаусса,калькулятор онлайн матриц методом гаусса,калькулятор онлайн матриц методом гаусса онлайн,калькулятор онлайн матриц методом гаусса онлайн калькулятор,калькулятор онлайн матрицы методом гаусса,калькулятор онлайн метод гаусса без дробей,калькулятор онлайн метод гаусса жордана гаусса онлайн калькулятор,калькулятор онлайн решение линейных уравнений,калькулятор онлайн решение матриц методом гаусса,калькулятор онлайн решение методом гаусса,калькулятор онлайн решение методом гаусса онлайн с подробным решением,калькулятор онлайн решение систем,калькулятор онлайн решение системы,калькулятор онлайн решение системы методом гаусса онлайн,калькулятор онлайн решение системы уравнений,калькулятор онлайн решить систему методом гаусса,калькулятор онлайн систем уравнений,калькулятор онлайн системы линейных уравнений,калькулятор онлайн системы линейных уравнений методом гаусса онлайн,калькулятор онлайн системы уравнений,калькулятор онлайн системы уравнений методом гаусса онлайн,калькулятор онлайн слау,калькулятор по методу гаусса,калькулятор решение линейных уравнений онлайн,калькулятор решение матриц методом гаусса,калькулятор решение методом гаусса,калькулятор решение методом гаусса онлайн,калькулятор решение систем линейных уравнений,калькулятор решение систем линейных уравнений методом гаусса,калькулятор решение систем методом гаусса,калькулятор решение систем методом гаусса онлайн,калькулятор решение систем уравнений методом гаусса,калькулятор решение систем уравнений методом гаусса онлайн,калькулятор решение системы методом гаусса,калькулятор решение системы уравнений,калькулятор решение системы уравнений методом гаусса,калькулятор решение слау методом гаусса,калькулятор решение уравнений методом гаусса,калькулятор решение уравнений методом гаусса онлайн,калькулятор решения линейных уравнений,калькулятор решения систем линейных уравнений,калькулятор решения уравнений линейных,калькулятор решить систему методом гаусса,калькулятор систем линейных уравнений,калькулятор систем линейных уравнений методом гаусса,калькулятор систем линейных уравнений онлайн,калькулятор систем онлайн,калькулятор систем уравнений онлайн,калькулятор систем уравнений с решением онлайн,калькулятор система линейных уравнений,калькулятор система уравнений,калькулятор системы линейных уравнений,калькулятор системы линейных уравнений онлайн,калькулятор системы уравнений,калькулятор системы уравнений онлайн,калькулятор системы уравнений онлайн с решением,калькулятор системы уравнений с решением онлайн,калькулятор системы уравнения,калькулятор слау,калькулятор слау методом гаусса,калькулятор слау онлайн,калькулятор слу,калькулятор уравнение линейное,линейное уравнение калькулятор,линейное уравнение калькулятор онлайн,линейное уравнение онлайн,линейное уравнение онлайн калькулятор,линейное уравнение онлайн решение,линейное уравнение решение онлайн,линейное уравнение решить онлайн,линейные уравнения калькулятор,линейные уравнения калькулятор онлайн,линейные уравнения онлайн калькулятор,линейные уравнения онлайн решать,линейные уравнения онлайн решение,линейные уравнения онлайн решить,линейные уравнения решать онлайн,линейные уравнения решение онлайн,матрица гаусса онлайн,матрица калькулятор метод гаусса,матрица калькулятор онлайн метод гаусса,матрица метод гаусса калькулятор,матрица метод гаусса онлайн,матрица метод гаусса онлайн калькулятор,матрица методом гаусса онлайн,матрица онлайн гаусса,матрица онлайн калькулятор метод гаусса,матрица онлайн метод гаусса,матрица онлайн методом гаусса,матрица онлайн решение методом гаусса,матрица расширенная онлайн,матрица решение методом гаусса онлайн,матрица решение онлайн методом гаусса,матрицы гаусса калькулятор,матрицы калькулятор гаусса,матрицы калькулятор метод гаусса,матрицы метод гаусса калькулятор,матрицы метод гаусса онлайн,матрицы метод гаусса онлайн калькулятор,матрицы метод гаусса онлайн калькулятор с подробным решением,матрицы методом гаусса калькулятор,матрицы методом гаусса калькулятор онлайн,матрицы методом гаусса онлайн,матрицы методом гаусса онлайн калькулятор,матрицы онлайн калькулятор метод гаусса,матрицы онлайн калькулятор методом гаусса,матрицы онлайн калькулятор с решением метод гаусса,матрицы онлайн метод гаусса,матрицы онлайн методом гаусса,матрицы решение гаусса онлайн,матричный калькулятор гаусса,матричный калькулятор метод гаусса,матричный калькулятор метод гаусса онлайн,матричный калькулятор методом гаусса,матричный калькулятор онлайн метод гаусса,матричный онлайн калькулятор метод гаусса,метод гаусса для матриц онлайн,метод гаусса жордана гаусса онлайн калькулятор,метод гаусса жордана калькулятор,метод гаусса жордана онлайн,метод гаусса жордана онлайн калькулятор с подробным решением,метод гаусса калькулятор,метод гаусса калькулятор матрицы,метод гаусса калькулятор онлайн,метод гаусса калькулятор онлайн с решением,метод гаусса калькулятор с решением,метод гаусса матриц онлайн калькулятор,метод гаусса матрица онлайн,метод гаусса матрица онлайн калькулятор,метод гаусса матрицы калькулятор,метод гаусса матрицы онлайн,метод гаусса матрицы онлайн калькулятор с подробным решением,метод гаусса матричный калькулятор,метод гаусса онлайн,метод гаусса онлайн калькулятор,метод гаусса онлайн калькулятор без дробей,метод гаусса онлайн калькулятор матриц,метод гаусса онлайн калькулятор с подробным,метод гаусса онлайн калькулятор с подробным решением,метод гаусса онлайн калькулятор с подробным решением и с проверкой,метод гаусса онлайн калькулятор с подробным решением матрицы,метод гаусса онлайн калькулятор с решением,метод гаусса онлайн матрица,метод гаусса онлайн матрицы,метод гаусса онлайн матричный метод,метод гаусса онлайн решение,метод гаусса онлайн решение матриц,метод гаусса онлайн решения,метод гаусса онлайн решить,метод гаусса онлайн с подробным решением,метод гаусса онлайн слау,метод гаусса примеры с решением онлайн,метод гаусса решение матриц онлайн,метод гаусса решение матриц онлайн калькулятор,метод гаусса решение онлайн,метод гаусса решение систем линейных уравнений онлайн,метод гаусса решения онлайн,метод гаусса решения систем линейных уравнений онлайн,метод гаусса решения систем линейных уравнений онлайн калькулятор,метод гаусса решить онлайн,метод гаусса с подробным решением калькулятор,метод гаусса с подробным решением онлайн,метод гаусса слау онлайн,метод жордана гаусса калькулятор,метод жордана гаусса онлайн,метод жордана гаусса онлайн калькулятор,метод жордана гаусса онлайн калькулятор с подробным решением,метод решение гаусса онлайн,метод решения гаусса онлайн,метод решения систем линейных уравнений метод гаусса онлайн,методом гаусса жордана онлайн,методом гаусса калькулятор,методом гаусса матрицы онлайн,методом гаусса найти общее решение системы линейных уравнений онлайн,методом гаусса онлайн калькулятор,методом гаусса решить систему калькулятор,методом гаусса решить систему линейных уравнений онлайн,методом жордана гаусса онлайн,найти матрицу методом гаусса онлайн,найти матрицу онлайн методом гаусса,найти общее решение системы линейных уравнений методом гаусса онлайн,найти общее решение системы линейных уравнений онлайн,найти общее решение системы линейных уравнений онлайн методом гаусса,найти определитель методом гаусса онлайн,найти определитель онлайн методом гаусса,найти решение системы линейных уравнений онлайн,онлайн гаусс,онлайн гаусса,онлайн калькулятор гаусса,онлайн калькулятор гаусса жордана гаусса онлайн,онлайн калькулятор жордан гаусс,онлайн калькулятор исследовать систему на совместность,онлайн калькулятор исследовать систему на совместность онлайн,онлайн калькулятор линейное уравнение,онлайн калькулятор линейных систем уравнений,онлайн калькулятор линейных уравнений,онлайн калькулятор линейных уравнений метод гаусса онлайн,онлайн калькулятор матриц гаусса,онлайн калькулятор матриц метод гаусса,онлайн калькулятор матриц метод гаусса с решением,онлайн калькулятор матриц методом гаусса,онлайн калькулятор матриц с решением метод гаусса,онлайн калькулятор матрица методом гаусса,онлайн калькулятор матрицы метод гаусса,онлайн калькулятор матрицы методом гаусса,онлайн калькулятор матрицы методом гаусса онлайн с решением,онлайн калькулятор матрицы с решением метод гаусса,онлайн калькулятор матричный метод гаусса,онлайн калькулятор метод гаусса,онлайн калькулятор метод гаусса без дробей,онлайн калькулятор метод гаусса матрицы,онлайн калькулятор метод гаусса с решением,онлайн калькулятор методом гаусса,онлайн калькулятор методом гаусса жордана гаусса онлайн,онлайн калькулятор методом гаусса решить систему,онлайн калькулятор методом гаусса решить систему уравнений,онлайн калькулятор решение линейных уравнений,онлайн калькулятор решение линейных уравнений методом гаусса,онлайн калькулятор решение матриц методом гаусса,онлайн калькулятор решение матрицы методом гаусса,онлайн калькулятор решение методом гаусса,онлайн калькулятор решение методом гаусса онлайн с подробным решением,онлайн калькулятор решение систем,онлайн калькулятор решение систем линейных уравнений,онлайн калькулятор решение систем линейных уравнений методом гаусса,онлайн калькулятор решение систем методом гаусса,онлайн калькулятор решение систем уравнений,онлайн калькулятор решение систем уравнений методом гаусса,онлайн калькулятор решение системы,онлайн калькулятор решение системы линейных уравнений,онлайн калькулятор решение системы линейных уравнений методом гаусса,онлайн калькулятор решение системы методом гаусса онлайн,онлайн калькулятор решение системы уравнений методом гаусса,онлайн калькулятор решение слау,онлайн калькулятор решение слау методом гаусса,онлайн калькулятор решение уравнений методом гаусса,онлайн калькулятор решение уравнений методом гаусса онлайн,онлайн калькулятор решения уравнений методом гаусса,онлайн калькулятор решите систему уравнений,онлайн калькулятор решить матрицу методом гаусса,онлайн калькулятор решить систему линейных уравнений методом гаусса,онлайн калькулятор решить систему методом гаусса,онлайн калькулятор решить систему уравнений методом гаусса,онлайн калькулятор решить уравнение методом гаусса,онлайн калькулятор систем,онлайн калькулятор систем линейных уравнений методом гаусса онлайн,онлайн калькулятор систем уравнений,онлайн калькулятор система линейных алгебраических уравнений,онлайн калькулятор система линейных уравнений,онлайн калькулятор система линейных уравнений методом гаусса,онлайн калькулятор система уравнений методом гаусса,онлайн калькулятор системы линейных уравнений,онлайн калькулятор системы линейных уравнений методом гаусса онлайн,онлайн калькулятор системы уравнений,онлайн калькулятор системы уравнений методом гаусса онлайн,онлайн калькулятор слау,онлайн калькулятор слау методом гаусса,онлайн калькулятор уравнение методом гаусса онлайн,онлайн линейное уравнение,онлайн линейные уравнения,онлайн матрица гаусса,онлайн матрица метод гаусса,онлайн матрица методом гаусса,онлайн матрицы метод гаусса,онлайн матрицы методом гаусса,онлайн матричный калькулятор метод гаусса,онлайн методом гаусса,онлайн решение гаусса,онлайн решение гауссом,онлайн решение задач методом гаусса,онлайн решение канонических уравнений,онлайн решение линейное уравнение,онлайн решение линейных уравнений,онлайн решение линейных уравнений методом гаусса,онлайн решение матриц гаусса,онлайн решение матриц метод гаусса,онлайн решение матриц методом гаусса,онлайн решение матриц методом гаусса жордана,онлайн решение матриц методом гаусса с решением,онлайн решение матриц по гауссу,онлайн решение матриц по методу гаусса,онлайн решение матрицы гаусса,онлайн решение матрицы метод гаусса,онлайн решение матрицы методом гаусса онлайн с решением,онлайн решение матричных уравнений методом гаусса,онлайн решение метод гаусса,онлайн решение методом гаусса,онлайн решение методом гаусса жордана,онлайн решение методом гаусса жордана гаусса,онлайн решение методом гаусса жордана онлайн,онлайн решение методом гаусса с подробным решением,онлайн решение методом жордана гаусса,онлайн решение систем,онлайн решение систем линейных алгебраических уравнений,онлайн решение систем линейных уравнений,онлайн решение систем методом гаусса,онлайн решение систем методом гаусса онлайн калькулятор,онлайн решение систем уравнений,онлайн решение систем уравнений методом гаусса,онлайн решение система линейных уравнений,онлайн решение систему уравнений,онлайн решение системы,онлайн решение системы линейных уравнений,онлайн решение системы линейных уравнений методом гаусса,онлайн решение системы методом гаусса,онлайн решение системы методом гаусса онлайн с,онлайн решение системы уравнений методом гаусса,онлайн решение системы уравнений методом гаусса онлайн с решением,онлайн решение системы уравнений с тремя неизвестными,онлайн решение системы уравнения,онлайн решение слау методом жордана гаусса,онлайн решение уравнений гаусса,онлайн решение уравнений методом гаусса,онлайн решение уравнений методом жордана гаусса онлайн,онлайн решение уравнений с тремя неизвестными,онлайн решение уравнения методом гаусса,онлайн решения матриц методом гаусса,онлайн решения метод гаусса,онлайн решения методом гаусса онлайн,онлайн решения систем уравнений,онлайн решить систему линейных уравнений методом гаусса,онлайн решить уравнение методом гаусса онлайн,онлайн система,онлайн система гаусса,онлайн система уравнений методом гаусса,онлайн система уравнений методом гаусса онлайн,онлайн система уравнений решение,онлайн системы,онлайн уравнение гаусса,посчитать матрицу методом гаусса онлайн,посчитать матрицу онлайн методом гаусса,проверить на совместимость матрицу онлайн,проверить на совместность систему онлайн,проверить систему на совместность онлайн,проверить совместимость системы уравнений онлайн,проверить совместность системы уравнений онлайн,проверка на совместность матрицы онлайн,расширенная матрица онлайн,решатель систем уравнений онлайн,решать онлайн линейные уравнения,решать онлайн систему уравнений,решение гаусса онлайн,решение гауссом онлайн,решение задач методом гаусса онлайн,решение канонических уравнений онлайн,решение линейное уравнение онлайн,решение линейные уравнения онлайн,решение линейных алгебраических уравнений онлайн,решение линейных систем уравнений калькулятор,решение линейных систем уравнений калькулятор онлайн,решение линейных уравнений калькулятор онлайн,решение линейных уравнений методом гаусса онлайн,решение линейных уравнений методом гаусса онлайн калькулятор,решение линейных уравнений онлайн,решение линейных уравнений онлайн калькулятор,решение линейных уравнений онлайн калькулятор с решением,решение линейных уравнений онлайн методом гаусса,решение матриц гаусса онлайн,решение матриц гауссом онлайн,решение матриц метод гаусса онлайн,решение матриц метод гаусса онлайн калькулятор,решение матриц методом гаусса жордана онлайн,решение матриц методом гаусса калькулятор,решение матриц методом гаусса онлайн,решение матриц методом гаусса онлайн калькулятор,решение матриц методом гаусса онлайн с подробным решением,решение матриц методом гаусса онлайн с решением,решение матриц методом гаусса онлайн с решением подробно,решение матриц методом жордана гаусса онлайн,решение матриц онлайн гаусса,решение матриц онлайн гауссом,решение матриц онлайн калькулятор метод гаусса,решение матриц онлайн калькулятор методом гаусса,решение матриц онлайн метод гаусса,решение матриц онлайн метод гаусса онлайн,решение матриц онлайн методом гаусса,решение матриц онлайн методом гаусса онлайн,решение матриц онлайн методом гаусса онлайн с,решение матриц онлайн методом гаусса с подробным решением,решение матриц онлайн методом гаусса с решением,решение матриц онлайн методом жордана гаусса,решение матриц онлайн по методу гаусса,решение матриц онлайн с подробным решением методом гаусса,решение матриц онлайн с решением методом гаусса,решение матриц по гауссу онлайн,решение матриц по методу гаусса онлайн,решение матрица методом гаусса онлайн,решение матрицы гаусса онлайн,решение матрицы методом гаусса онлайн,решение матрицы методом гаусса онлайн калькулятор,решение матрицы методом гаусса онлайн решение,решение матрицы методом гаусса онлайн с подробным решением,решение матрицы методом гаусса онлайн с решением,решение матрицы методом гаусса онлайн с решением калькулятор,решение матрицы онлайн гаусса,решение матрицы онлайн методом гаусса,решение матрицы онлайн методом гаусса онлайн,решение матрицы онлайн методом гаусса с подробным решением,решение матрицы онлайн методом гаусса с решением,решение матричных уравнений методом гаусса онлайн,решение матричных уравнений онлайн методом гаусса,решение метод гаусса онлайн,решение методом гаусса жордана онлайн,решение методом гаусса калькулятор,решение методом гаусса калькулятор онлайн,решение методом гаусса матрицы онлайн калькулятор,решение методом гаусса онлайн,решение методом гаусса онлайн калькулятор,решение методом гаусса онлайн с подробным решением,решение методом гаусса онлайн с решением,решение методом жордана гаусса онлайн,решение онлайн гаусса,решение онлайн гауссом,решение онлайн линейные уравнения,решение онлайн линейных уравнений методом гаусса,решение онлайн метод гаусса,решение онлайн методом гаусса,решение онлайн методом гаусса с подробным решением,решение онлайн методом жордана гаусса,решение онлайн систем методом гаусса онлайн калькулятор,решение онлайн система линейных уравнений,решение онлайн система уравнений,решение онлайн системы линейных уравнений методом гаусса,решение онлайн системы методом гаусса онлайн с,решение онлайн уравнений с 3 неизвестными,решение по методу гаусса онлайн,решение расширенной матрицы онлайн,решение систем калькулятор онлайн,решение систем линейных алгебраических уравнений онлайн,решение систем линейных уравнений калькулятор,решение систем линейных уравнений калькулятор онлайн,решение систем линейных уравнений метод гаусса онлайн,решение систем линейных уравнений методом гаусса калькулятор,решение систем линейных уравнений методом гаусса онлайн,решение систем линейных уравнений методом гаусса онлайн калькулятор,решение систем линейных уравнений методом гаусса онлайн с решением,решение систем линейных уравнений онлайн,решение систем линейных уравнений онлайн калькулятор,решение систем линейных уравнений онлайн метод гаусса,решение систем линейных уравнений онлайн с решением,решение систем методом гаусса калькулятор,решение систем методом гаусса калькулятор онлайн,решение систем методом гаусса онлайн,решение систем методом гаусса онлайн калькулятор,решение систем онлайн калькулятор,решение систем онлайн методом гаусса,решение систем онлайн с решением,решение систем уравнений калькулятор онлайн,решение систем уравнений методом гаусса калькулятор,решение систем уравнений методом гаусса онлайн,решение систем уравнений методом гаусса онлайн калькулятор,решение систем уравнений методом гаусса онлайн с подробным решением,решение систем уравнений онлайн,решение систем уравнений онлайн калькулятор,решение систем уравнений онлайн метод гаусса онлайн,решение систем уравнений онлайн методом гаусса,решение систем уравнений онлайн с подробным решением,решение систем уравнений онлайн с подробным решением методом гаусса,решение систем уравнения онлайн,решение система линейных уравнений онлайн,решение система уравнений онлайн,решение системных уравнений методом гаусса онлайн,решение системных уравнений онлайн методом гаусса,решение систему уравнений онлайн,решение системы линейных уравнений калькулятор онлайн,решение системы линейных уравнений методом гаусса онлайн,решение системы линейных уравнений методом гаусса онлайн калькулятор,решение системы линейных уравнений методом гаусса онлайн с решением,решение системы линейных уравнений онлайн,решение системы линейных уравнений онлайн калькулятор,решение системы линейных уравнений онлайн методом гаусса,решение системы методом гаусса калькулятор,решение системы методом гаусса онлайн,решение системы методом гаусса онлайн с решением,решение системы онлайн,решение системы онлайн калькулятор,решение системы онлайн методом гаусса,решение системы онлайн методом гаусса онлайн с,решение системы уравнений методом гаусса калькулятор,решение системы уравнений методом гаусса калькулятор онлайн,решение системы уравнений методом гаусса онлайн,решение системы уравнений методом гаусса онлайн калькулятор,решение системы уравнений методом гаусса онлайн с решением,решение системы уравнений онлайн,решение системы уравнений онлайн калькулятор,решение системы уравнений онлайн методом гаусса,решение системы уравнений онлайн методом гаусса онлайн,решение системы уравнений онлайн с решением,решение системы уравнений с тремя неизвестными онлайн,решение системы уравнения онлайн,решение слау калькулятор онлайн,решение слау методом гаусса жордана онлайн,решение слау методом гаусса калькулятор,решение слау методом гаусса онлайн,решение слау методом гаусса онлайн калькулятор,решение слау методом жордана гаусса онлайн,решение слау онлайн,решение слау онлайн калькулятор,решение слау онлайн методом гаусса,решение слау онлайн методом гаусса онлайн,решение слау онлайн методом жордана гаусса,решение слу метод гаусса онлайн,решение слу онлайн,решение слу онлайн метод гаусса,решение уравнений гаусса онлайн,решение уравнений методом гаусса жордана гаусса онлайн,решение уравнений методом гаусса жордана онлайн,решение уравнений методом гаусса калькулятор,решение уравнений методом гаусса калькулятор онлайн,решение уравнений методом гаусса онлайн,решение уравнений методом гаусса онлайн калькулятор,решение уравнений методом гаусса онлайн с решением,решение уравнений методом гаусса решение онлайн калькулятор,решение уравнений методом жордана гаусса онлайн,решение уравнений онлайн гаусса,решение уравнений онлайн методом гаусса,решение уравнений онлайн методом гаусса калькулятор онлайн,решение уравнений онлайн с 3 неизвестными,решение уравнений с 3 неизвестными онлайн,решение уравнения методом гаусса онлайн,решение уравнения онлайн методом гаусса,решения линейных уравнений калькулятор,решения матриц методом гаусса онлайн,решения матриц онлайн методом гаусса,решения онлайн методом гаусса онлайн,решения систем уравнений методом гаусса калькулятор,решите линейное уравнение онлайн,решите систему уравнений методом гаусса онлайн,решите систему уравнений онлайн с решением,решить линейное уравнение методом гаусса онлайн,решить линейное уравнение онлайн,решить линейное уравнение онлайн методом гаусса,решить матрицу методом гаусса онлайн,решить матрицу методом гаусса онлайн калькулятор,решить матрицу методом гаусса онлайн с подробным решением,решить матрицу методом гаусса онлайн с решением,решить матрицу онлайн калькулятор методом гаусса,решить матрицу онлайн методом гаусса,решить матрицу онлайн методом гаусса онлайн,решить матрицу онлайн методом гаусса онлайн с,решить матрицу онлайн методом гаусса с решением,решить метод гаусса онлайн,решить методом гаусса онлайн,решить методом гаусса онлайн с подробным решением,решить методом гаусса систему линейных алгебраических уравнений онлайн,решить методом гаусса систему линейных уравнений онлайн,решить методом гаусса слау онлайн,решить неоднородную систему линейных уравнений методом гаусса,решить неоднородную систему линейных уравнений методом гаусса онлайн,решить онлайн алгебраическое уравнение,решить онлайн линейные уравнения,решить онлайн матрицу методом гаусса,решить онлайн метод гаусса,решить онлайн методом гаусса,решить онлайн систему линейных уравнений методом гаусса,решить онлайн систему уравнение,решить онлайн систему уравнений с решением,решить онлайн системы уравнений,решить онлайн уравнение методом гаусса,решить систему линейных алгебраических уравнений методом гаусса онлайн,решить систему линейных уравнений методом гаусса калькулятор онлайн,решить систему линейных уравнений методом гаусса онлайн,решить систему линейных уравнений методом гаусса онлайн калькулятор,решить систему линейных уравнений методом гаусса онлайн с решением,решить систему линейных уравнений онлайн,решить систему линейных уравнений онлайн методом гаусса,решить систему методом гаусса жордана онлайн,решить систему методом гаусса калькулятор,решить систему методом гаусса калькулятор онлайн,решить систему методом гаусса онлайн,решить систему методом гаусса онлайн калькулятор,решить систему методом гаусса онлайн с подробным решением,решить систему методом жордана гаусса онлайн,решить систему уравнение онлайн с решением,решить систему уравнений калькулятор онлайн,решить систему уравнений калькулятор онлайн с решением,решить систему уравнений методом гаусса калькулятор онлайн,решить систему уравнений методом гаусса онлайн,решить систему уравнений методом гаусса онлайн калькулятор,решить систему уравнений методом гаусса онлайн с подробным решением,решить систему уравнений методом гаусса онлайн с подробным решением онлайн,решить систему уравнений онлайн калькулятор с решением,решить систему уравнений онлайн методом гаусса,решить систему уравнений онлайн методом гаусса онлайн,решить систему уравнений онлайн с комплексными числами,решить систему уравнений онлайн с подробным решением,решить систему уравнений онлайн с решением,решить систему уравнений с комплексными числами онлайн,решить систему уравнений с тремя неизвестными онлайн,решить систему уравнения онлайн,решить системы линейных уравнений методом гаусса онлайн,решить системы уравнений онлайн,решить слау,решить слау методом гаусса онлайн,решить слау методом гаусса онлайн с решением,решить слау онлайн,решить слау онлайн методом гаусса,решить уравнение методом гаусса онлайн,решить уравнение методом гаусса онлайн калькулятор,решить уравнение онлайн методом гаусса,решить уравнение онлайн методом гаусса онлайн,решить уравнение с тремя неизвестными онлайн,систем линейных уравнений методом гаусса калькулятор,систем линейных уравнений онлайн калькулятор,система гаусса онлайн,система линейных алгебраических уравнений онлайн калькулятор,система линейных уравнений калькулятор,система линейных уравнений калькулятор онлайн,система линейных уравнений методом гаусса калькулятор онлайн,система линейных уравнений методом гаусса онлайн,система линейных уравнений методом гаусса онлайн калькулятор,система линейных уравнений онлайн,система линейных уравнений онлайн калькулятор,система линейных уравнений онлайн методом гаусса,система линейных уравнений онлайн решение,система линейных уравнений решение онлайн,система методом гаусса онлайн,система уравнений гаусса онлайн,система уравнений калькулятор,система уравнений методом гаусса онлайн,система уравнений методом гаусса онлайн калькулятор,система уравнений онлайн гаусса,система уравнений онлайн калькулятор,система уравнений онлайн калькулятор с подробным решением,система уравнений онлайн методом гаусса,система уравнений онлайн методом гаусса онлайн,система уравнений онлайн решение,системы линейных алгебраических уравнений онлайн,системы линейных уравнений калькулятор онлайн,системы линейных уравнений онлайн,системы линейных уравнений онлайн калькулятор,системы онлайн калькулятор,системы уравнений калькулятор,системы уравнений калькулятор онлайн,системы уравнений методом гаусса калькулятор,системы уравнений онлайн,системы уравнений онлайн калькулятор,слау калькулятор,слау калькулятор онлайн,слау метод гаусса онлайн,слау методом гаусса жордана гаусса онлайн,слау методом гаусса калькулятор,слау методом гаусса онлайн,слау методом гаусса онлайн калькулятор,слау онлайн,слау онлайн калькулятор,слау онлайн метод гаусса,слу калькулятор,слу калькулятор онлайн,слу онлайн калькулятор,слу онлайн решение,слу решить,совместность матрицы онлайн,уравнение гаусса онлайн,уравнение методом гаусса онлайн,уравнение с тремя неизвестными онлайн,уравнения онлайн методом гаусса онлайн.
На этой странице вы найдёте калькулятор, который поможет решить любой вопрос, в том числе и вычисление матрицы онлайн методом гаусса онлайн. Просто введите задачу в окошко и нажмите «решить» здесь (например, гаусс калькулятор).

Где можно решить любую задачу по математике, а так же вычисление матрицы онлайн методом гаусса онлайн Онлайн?

Решить задачу вычисление матрицы онлайн методом гаусса онлайн вы можете на нашем сайте https://pocketteacher.ru. Бесплатный онлайн решатель позволит решить онлайн задачу любой сложности за считанные секунды. Все, что вам необходимо сделать — это просто ввести свои данные в решателе. Так же вы можете посмотреть видео инструкцию и узнать, как правильно ввести вашу задачу на нашем сайте. А если у вас остались вопросы, то вы можете задать их в чате снизу слева на странице калькулятора.

Решение системы линейных уравнений методом Гаусса онлайн

Для решения любой системы линейных уравнений метод Гаусса или метод последовательного исключения неизвестных является наиболее универсальным и достаточно простым при небольшом количестве переменных. Этот метод универсален, его применяют, когда система уравнений имеет:

  • единственное решение;
  • бесконечное множество решений;
  • вовсе не имеет решений.

Суть метода состоит в переходе от заданной системы линейных уравнений к более простой с помощью таких эквивалентных преобразований в системе, как:

  • перемена двух уравнений местами;
  • умножение обеих частей уравнения на любое действительное число, не равное 0;
  • прибавление к одному уравнению соответствующих частей другого, умноженных на произвольное число.

С помощью преобразований последовательно исключаем одну переменную за другой пока в одной из строк не будет определена переменная xi.

Метод Гаусса позволяет решать СЛАУ при небольшом числе вычислительных операций.

Алгоритм решения:

  • записываем систему в виде расширенной матрицы;
  • прямой ход — приводим матрицу к ступенчатому виду;
  • обратный ход — приводим матрицу к специальному ступенчатому виду.

Пусть дана система из n уравнений с n неизвестными переменными:

Определитель основной матрицы не равен 0.

Исключим из всех уравнений системы переменную х1, начиная со 2-го, для чего:

  • ко 2-му уравнению прибавим 1-е, умноженное на — а2111;
  • к 3-му уравнению прибавим 1-е, умноженное на — а3111, и т.д.;
  • к n-му уравнению прибавим 1-е, умноженное на — аn111.

В результате преобразований система приняла вид:

Далее таким же путем исключаем неизвестную переменную х2 из всех уравнений, начиная с 3-го.

Для этого к 3-му уравнению прибавляем 2-е, умноженное на — а3222 и т.д. К n-му уравнению прибавим 2-е, умноженное на — аn222.

Таким же способом исключаем неизвестную х3 из всех уравнений системы, начиная с 4-го.

Прямой ход продолжается, пока в последнем уравнении не останется единственная неизвестная. Система будет иметь вид:

аnn(n-1) хn = bn(n-1)

После окончания прямого хода метода Гаусса — последовательного исключения неизвестных, вычисляем неизвестную в последнем уравнении:

  • из последнего уравнения системы находим хn по формуле:
  • из предпоследнего уравнения находим хn-1 и т.д.
  • из первого уравнения находим х1.

Последовательное нахождение неизвестных, начиная с последнего уравнения к первому, называется обратным ходом.

Заметим, если в матрице есть хоть одна нулевая строка, у которой правая часть (свободный член) не равна 0, система несовместима, решения отсутствуют.

Для быстрого и правильного решения СЛАУ методом Гаусса можно воспользоваться калькулятором онлайн.


Решение системы линейных уравнений методом Гаусса
123456  — количество неизвестных
Количество знаков после разделителя дроби в числах: 0123456789101112

Решение уравнений методом Гаусса | matematicus.

ru

С помощью метода Гаусса можно решить любую систему линейных уравнений с различным числом уравнений и неизвестных переменных. И именно этим свойством этот метод превосходит матричный метод и метод Крамера.

Суть метода состоит в приведении системы линейных уравнений к ступенчатому (треугольному) виду за счет последовательного исключения неизвестных. Затем её решения с помощью обратной подстановки.


Допустимые преобразования матрицы:

  1. Перестановка местами двух строк или двух столбцов;
  2. Умножение строки на число, которое не равно 0;
  3. Прибавление одной строки к другой.
  4. Исключение или добавление нулевой строки

Допустим, дана система линейных алгебраических уравнений с четырьмя уравнениями и четырьмя неизвестными.

Составим расширенную матрицу СЛАУ:

Затем первое уравнение СЛАУ делим на a11.  При этом a11≠0, если равно нуля, то переставляем две строки или два столбца местами так, чтобы избавится от нуля. После полученное уравнение умножаем на a21 и вычитаем из второго уравнения, дальше, умножаем на a31 и вычитаем из третьего уравнения и т.д.

Также поступаем и с оставшемся уравнениями, т.е. со вторым, третьем и четвёртым. В итоге должна получится матрица ступенчатого или треугольного вида.

Система уравнений примет вид

Такую систему элементарно решить обратным ходом, т.е. последовательным решением уравнений от нижнего к верхнему.

Рассмотрим наиболее подробно метод Гаусса при решении СЛАУ на практике.

Пример 1

Решить методом Гаусса систему уравнений

Решение

Составим расширенную матрицу системы уравнений:

Первую строку разделим на a11, но так как в этой строке a11=0, то необходимо поменять строку у которой первый элемент не равен нулю. Выберем по модулю наибольшей элемент, это a41=2 Поэтому поменяем первую и четвёртую строки местами.

Получаем:

Первую строку разделим на a11=2. Получим матрицу:

Умножаем элементы первой строки на -1 и прибавляем к элементам второй строк. Получим матрицу:

Умножаем элементы первой строки на -1 и прибавляем к элементам третьей строки.

Четвёртую строку оставляем без изменений, так как её первый элемент равен нулю.

Теперь первый столбец не трогаем.

Начинаем повторять действия, которые применяли ранее.

Второе уравнение разделим на a22=-1/2, тогда

Умножаем элементы второй строки на -1/2 и прибавляем к элементам третьей строки.

Умножаем элементы второй строки на -1 и прибавляем к элементам четвёртой строки.

Первый и второй столбец не трогаем.

Третьей столбец разделим на 2.

Умножаем элементы третьей строки на -1 и прибавляем к элементам четвёртой строки.

Получаем ступенчатую систему алгебраических уравнений:

Отсюда, решая систему снизу вверх, получаем корни системы уравнений


Приведём простой пример краткой записи решения СЛАУ методом Гаусса

Пример 2

Решить систему линейных уравнений с тремя неизвестными методом Гаусса.

Решение

Составим расширенную матрицу системы линейных уравнений .

Следовательно, искомая система может быть представлена в ступенчатом виде:

Решая последовательно уравнение, получаем:

Ответ: z = 3; y = 2; x = 1

Численные методы: решение систем линейных уравнений

В прикладных задачах часто возникает необходимость решать системы линейных уравнений.

Система линейных алгебраических уравнений с n неизвестными  —  это система уравнений вида

                                     (1)

Слово система означает, что все уравнения рассматриваются как одно целое.

В общем случае у нас имеется m — уравнений, n — количество неизвестных. x1x2,…, xn — неизвестные, которые следует определить.

В системе (1)  – фиксированные коэффициенты,  b1b2, …, bm — свободные члены — предполагаются известными.

Система (1) называется однородной, если все её свободные члены равны нулю (b1 = b2 = … = bm = 0), иначе — неоднородной.

Система (1) называется квадратной, если число m уравнений равно числу n неизвестных.

Задача состоит в том, чтобы найти такие  которые удовлетворяют всем уравнениям (1).

В частном случае мы имеем одно линейное уравнение:

Конечно, такое уравнение легко решить, если предположить, что коэффициент  не равен 0, имеем:  = .

Очевидно, в общем случае имеются 3 варианта решений: система имеет ни одного решения, имеет одно решение, более одного решения.

Система (1) называется совместной, если она имеет хотя бы одно решение, и несовместной, если нет ни одного решения.

Система линейных уравнений может быть представлена в матричной форме как:

или:

Ax = b

Здесь A — это матрица системы, x — столбец неизвестных, а b — столбец свободных членов.

Если к матрице A приписать справа столбец свободных членов, то получившаяся матрица называется расширенной.

Рассмотрим, например, систему вида и поймем, как найти ее решение:

                                      (2)

Предположим на минуту, что в первом уравнении y отсутствует, а во втором отсутствует x, тогда мы имели бы решение именно то решение, которое нам нужно.

Вопрос: как исходную систему привести к такому виду и можно ли это сделать.

Заметим, что с тождествами мы можем делать следующие вещи: домножать на одно и то же число, отличное от 0, складывать, вычитать и тд, это похоже с тем, что вы раскладываете монеты по своим карманам, не меняя общей суммы.

От этих операций тождество не меняется.

В системе (2) у нас два тождества, домножим второе тождество на 2 и вычтем из первого, получим:

                                      (3)

Формально у нас есть еще старое тождество , но оно нам не понадобится (подумайте, почему).

Система (3) точно такая же, как система (2).

Из второго уравнения системы (3) сразу получим:

 

Никто не мешает нам подставить это значение в первое уравнение:

Отсюда сразу находим, что

Итак, путем простых действий мы нашли, что система (2) может быть представлена в виде:

Именно такие естественные соображения приводят к общему методу решения систем линейных уравнений, известному как метод исключения или метод Гаусса.

Метод Гаусса является одним из самых распространенных прямых методов решения систем линейных уравнений Ax = b:

Опишем этот метод в общем случае.

Вначале исходная система приводится к верхнетреугольному виду.

Это достигается следующей последовательностью преобразований (прямой ход).

Будем считать для удобства, что элемент aij исходной матрицы и компоненты вектора bi есть, соответственно, элементы aij (1) первого шага преобразованной матрицы A1 и преобразованного вектора b1:A = A1, b=b1

Далее, на втором шаге прибавим к второй строке первую, умноженную на  

Аналогично поступим со всеми оставшимися строками, т.е. прибавим к каждой i-ой строке i=2,3,…,N, первую, умноженную на коэффициент  

При этом соответственно изменится и вектор b1.  

Таким образом, 2 шаг.

Имеем систему уравнений A2x = b2:

где

3 шаг.

Прибавим к новой третьей строке новую вторую, умноженную на  

То же самое сделаем с остальными строками 4,5,…,N, т.е. прибавим к i-ой строке вторую, умноженную на  

При этом получим систему A3x = b3:

(k+1)-ый шаг:

Здесь

Поступая так и далее, на шаге N-1 получаем верхнетреугольную систему:

При этом, мы также получили матрицу C переводных коэффициентов, имеющую вид:

Решение полученной треугольной системы  как легко видеть, имеет вид (обратный ход метода Гаусса):

Заметим, что при прямом ходе метода Гаусса может возникнуть ситуация, когда происходит деление на нуль, да и вообще, желательно не делить на малое число, чтобы не накапливалась ошибка.

Поэтому метод Гаусса обычно проводят с частичным выбором главного элемента, то есть после каждого шага (пусть это был k-й шаг) переставляют строки с номерами k,k+1,…,N таким образом, чтобы на месте kk оказался элемент  наибольший из всех в k-ом столбце при m>k (при этом, естественно, переставляются и компоненты вектора b).

Можно для максимальной точности переставлять также и столбцы преобразуемой матрицы, чтобы на месте kk оказался максимальный элемент из всех с индексами больше, либо равными k.

Эта процедура называется методом Гаусса с выбором главного элемента. Она несколько повышает точность по сравнению с частичным выбором главного элемента, но весьма неудобна, в том числе для программирования, поскольку при перестановке строк компоненты искомого вектора x переставлять не надо, тогда как при перестановке столбцов надо переставлять и соответствующие компоненты вектора x.

Опишем обратный ход метода Гаусса в несколько иной форме (треугольное разложение).

Введем матрицы Mk по правилу:

На каждом шаге метода Гаусса получается некоторая промежуточная матрица: 

 и вектор  

Нетрудно видеть, что

Вопрос. Почему

Если производить также выбор главных элементов, то необходимо использовать оператор P перестановки индексов l и m, матричные элементы которого равны:

При применении оператора перестановки индексов к матрице слева, меняются местами строки матрицы и компоненты свободного вектора (PAx = Pb), если же его применить справа к матрице, то меняются местами ее столбцы и компоненты решения

Существует большой класс так называемых итерационных методов решения систем уравнений, аналогичных итерационным методам нахождения корней нелинейных уравнений.

Итерационные методы последовательно уточняют решение, отправляясь от начального приближения.

При выполнении условий сходимости они позволяют достичь любой точности просто повторением итераций.

Преимущество этих методов в том, что часто они позволяют достичь решения с заранее заданной точностью быстрее, а также позволяют решать большие системы уравнений.

Идея состоит в том, чтобы найти неподвижную точку матричного уравнения

                                     (5)

эквивалентного начальной системе линейных алгебраических уравнений.

При итерации  в правой части уравнения заменяется, например, в методе Якоби (метод простой итерации) приближение, найденное на предыдущем шаге:

.

Термин неподвижная точка становится ясен, если вы внимательно посмотрите на уравнение (5), по самому своему смыслу величина Х является неподвижной точкой.

Более подробное описание методов решения систем линейных уравнений можно найти в специальной литературе, наша задача дать обзор методов и основные идеи решения такого рода задач.

Обусловленность линейных систем, погрешность

При решении абстрактной задачи Ax = b, где A — оператор произвольной природы, важным моментом является корректность ее постановки.

Задача считается корректной, если решение существует и единственно и , кроме того, решение непрерывно зависит от данных (то есть, при  также стремится к нулю).

Однако и непрерывная зависимость от входных данных может иметь свои нюансы.

Чем меньшее (большее) изменение решения вызывает вариация входных данных, тем более хорошо (плохо) обусловленной считается задача.

Понятие обусловленности является тем более существенным для численных методов, поскольку на практике входные данные известны, как правило, с некоторой погрешностью.

Кроме того, существуют ошибки округления, возникающие при вычислениях.

Таким образом, формально корректная задача, являясь плохо обусловленной, может оказаться разрешимой столь неточно, что в этом будет отсутствовать практический смысл.

Чем можно охарактеризовать количественно обусловленность для линейных систем?

Пусть A — квадратная NxN — матрица.

Рассмотрим задачу Ax = b.

Пусть также  некоторая норма в пространстве RN 

Норма оператора A определяется стандартно:

Обозначим y = Ax и введем число m по правилу:

Величина  называется числом обусловленности.

Очевидно:

  1.      
  2. если A — диагональная, то  (Для какой нормы, или для всех вышеприведенных?). Чем меньше число обусловленности C(A), тем лучше обусловлена система. Действительно, пусть  вариация правой части, а соответствующее изменение решения.

Тогда справедливо следующее неравенство:

 

Доказательство. Имеем:

Так как

то    

Аналогично, поскольку  

Объединяя два неравенства, окончательно получаем для оценки погрешности:

 

В начало

Содержание портала

Решение системы линейных уравнений методом Гаусса-Жордана

метод Гаусса–Жордана – один из наиболее известных и широко применяемых методов решения систем линейных уравнений. Матричный метод и метод Крамера обладают тем недостатком,
что они не дают ответа в том случае, когда detA = 0, а определяют лишь единственное решение при detA неравном 0. Еще одним недостатком является то, что объем математических вычислений
в рамках этих методов резко возрастает с ростом числа уравнений. Метод Гаусса практически свободен от этих недостатков.

Алгоритм метода Гаусса

  1. На основании системы линейных уравнений составляем расширенную матрицу системы;
  2. Приводим матрицу к “треугольному” виду;
  3. Определяем ранги основной и расширенной матриц, и на основании этого делаем вывод о совместности системы и количестве допустимых решений;
  4. В случае, если система имеет единственное решение производим обратную подстановку и находим его, если система имеет множество решений: выражаем базисные переменные через
    переменные которые могут принимать произвольные значения;

Комментарий к шагу 2 Метода Гаусса. Треугольной называют матрицу, в которой все элементы расположенные ниже главной диагонали равны нулю.

Для приведения исходной расширенной матрицы к треугольному виду используем следующие два свойства определителей:

Свойство 1. Определитель не изменит свое значение, если ко всем элементам какой-либо строки (столбца) матрицы прибавить соответствующие элементы параллельной строки (столбца), умноженные на произвольное одно и то же число.

Свойство 2. При перестановке двух любых столбцов или строк матрицы ее определитель меняет знак на противоположный, а абсолютная величина определителя остается неизменной.

На основании этих свойств определителей составим алгоритм преобразования матрицы к треугольному виду:

  1. Рассматриваем строку i(начиная с первой). Если, элемент aii равен нулю, меняем местами i-ю и i+1-ю строки матрицы. Знак определителя при этом изменится на противоположный. Если a11 отличен от нуля – переходим к следующему шагу;
  2. Для каждой строки j, ниже i-й находим значение коэффициента Kj=aji/aii;
  3. Пересчитываем элементы всех строк j, расположенных ниже текущей строки i, с использованием соответствующих коэффициентов по формуле: ajkнов.=ajk-Kj*aik;
    После чего, возвращаемся к первому шагу алгоритма и рассматриваем следующую строку, пока не доберемся до строки i=n-1, где n – размерность матрицы A
  4. В полученной треугольной матрице расчитываем произведение всех элементов главной диагонали Пaii, которое и будет являтся определителем;

Другими словами, суть метода можно сформулировать следующим образом. Нам необходимо сделать нулевыми все элементы матрицы ниже главной диагонали. Сначала мы получаем нули в первом столбце.
Для этого мы последовательно вычитаем первую строку, домноженную на нужное нам число (такое, чтоб при вычитании мы получили ноль в первом элементе строки), из всех ниже лежащих строк.
Затем проделываем то же самое для второй строки, чтобы получить нули во втором столбце ниже главной диагонали матрицы. И так далее пока не доберемся до предпоследней строки.

Комментарий к шагу 3 Метода Гаусса. Рангом матрицы A размера m × n называется наивысший порядок отличного от нуля минора этой матрицы. Ранг матрицы A обозначается через r(A) = rangA = rankA.
Минором M (от латинского “minor” меньший) k-го порядка матрицы A называется определитель некоторой матрицы, составленной из элементов матрицы A, стоящих на пересечении произвольно выбранных k
строк и k столбцов с сохранением их порядка. Если номера столбцов, в которых расположен минор M, совпадают с номерами строк, то этот минор называется главным. Каждая матрица A порядка n имеет
(Ckn)2 миноров k-го порядка. Минорами 1-го порядка являются сами элементы матрицы A.

Основываясь на сравнении полученных значений рангов для основной и расширенной матрицы можно сделать следующие выводы о разрешимости системы:

  • если ранг основной системы равен рангу расширенной и равен числу уравнений системы (rangA=rangA’=n), то система совместна и имеет единственное решение;
  • если ранг основной системы равен рангу расширенной, но меньше числа уравнений в системе (rangA=rangA’
  • если ранг основной системы меньше ранга расширенной (rangA

Решение уравнений в Excel методом итераций Крамера и Гаусса

В программе Excel имеется обширный инструментарий для решения различных видов уравнений разными методами.

Рассмотрим на примерах некоторые варианты решений.

Решение уравнений методом подбора параметров Excel

Инструмент «Подбор параметра» применяется в ситуации, когда известен результат, но неизвестны аргументы. Excel подбирает значения до тех пор, пока вычисление не даст нужный итог.

Путь к команде: «Данные» — «Работа с данными» — «Анализ «что-если»» — «Подбор параметра».

Рассмотрим на примере решение квадратного уравнения х2 + 3х + 2 = 0. Порядок нахождения корня средствами Excel:

  1. Введем в ячейку В2 формулу для нахождения значения функции. В качестве аргумента применим ссылку на ячейку В1.
  2. Открываем меню инструмента «Подбор параметра». В графе «Установить в ячейку» — ссылка на ячейку В2, где находится формула. В поле «Значение» вводим 0. Это то значение, которое нужно получить. В графе «Изменяя значение ячейки» — В1. Здесь должен отобразиться отобранный параметр.
  3. После нажатия ОК отобразится результат подбора. Если нужно его сохранить, вновь нажимаем ОК. В противном случае – «Отмена».

Для подбора параметра программа использует циклический процесс. Чтобы изменить число итераций и погрешность, нужно зайти в параметры Excel. На вкладке «Формулы» установить предельное количество итераций, относительную погрешность. Поставить галочку «включить итеративные вычисления».



Как решить систему уравнений матричным методом в Excel

Дана система уравнений:

  1. Значения элементов введем в ячейки Excel в виде таблицы.
  2. Найдем обратную матрицу. Выделим диапазон, куда впоследствии будут помещены элементы матрицы (ориентируемся на количество строк и столбцов в исходной матрице). Открываем список функций (fx). В категории «Математические» находим МОБР. Аргумент – массив ячеек с элементами исходной матрицы.
  3. Нажимаем ОК – в левом верхнем углу диапазона появляется значение. Последовательно жмем кнопку F2 и сочетание клавиш Ctrl + Shift + Enter.
  4. Умножим обратную матрицу Ах-1х на матрицу В (именно в таком порядке следования множителей!). Выделяем диапазон, где впоследствии появятся элементы результирующей матрицы (ориентируемся на число строк и столбцов матрицы В). Открываем диалоговое окно математической функции МУМНОЖ. Первый диапазон – обратная матрица. Второй – матрица В.
  5. Закрываем окно с аргументами функции нажатием кнопки ОК. Последовательно нажимаем кнопку F2 и комбинацию Ctrl + Shift + Enter.

Получены корни уравнений.

Решение системы уравнений методом Крамера в Excel

Возьмем систему уравнений из предыдущего примера:

Для их решения методом Крамера вычислим определители матриц, полученных заменой одного столбца в матрице А на столбец-матрицу В.

Для расчета определителей используем функцию МОПРЕД. Аргумент – диапазон с соответствующей матрицей.

Рассчитаем также определитель матрицы А (массив – диапазон матрицы А).

Определитель системы больше 0 – решение можно найти по формуле Крамера (Dx / |A|).

Для расчета Х1: =U2/$U$1, где U2 – D1. Для расчета Х2: =U3/$U$1. И т.д. Получим корни уравнений:

Решение систем уравнений методом Гаусса в Excel

Для примера возьмем простейшую систему уравнений:

3а + 2в – 5с = -1
2а – в – 3с = 13
а + 2в – с = 9

Коэффициенты запишем в матрицу А. Свободные члены – в матрицу В.

Для наглядности свободные члены выделим заливкой. Если в первой ячейке матрицы А оказался 0, нужно поменять местами строки, чтобы здесь оказалось отличное от 0 значение.

  1. Приведем все коэффициенты при а к 0. Кроме первого уравнения. Скопируем значения в первой строке двух матриц в ячейки В6:Е6. В ячейку В7 введем формулу: =B3:Е3-$B$2:$Е$2*(B3/$B$2). Выделим диапазон В7:Е7. Нажмем F2 и сочетание клавиш Ctrl + Shift + Enter. Мы отняли от второй строки первую, умноженную на отношение первых элементов второго и первого уравнения.
  2. Копируем введенную формулу на 8 и 9 строки. Так мы избавились от коэффициентов перед а. Сохранили только первое уравнение.
  3. Приведем к 0 коэффициенты перед в в третьем и четвертом уравнении. Копируем строки 6 и 7 (только значения). Переносим их ниже, в строки 10 и 11. Эти данные должны остаться неизменными. В ячейку В12 вводим формулу массива.
  4. Прямую прогонку по методу Гаусса сделали. В обратном порядке начнем прогонять с последней строки полученной матрицы. Все элементы данной строки нужно разделить на коэффициент при с. Введем в строку формулу массива: {=B12:E12/D12}.
  5. В строке 15: отнимем от второй строки третью, умноженную на коэффициент при с второй строки ({=(B11:E11-B16:E16*D11)/C11}). В строке 14: от первой строки отнимаем вторую и третью, умноженные на соответствующие коэффициенты ({=(B10:E10-B15:E15*C10-B16:E16*D10)/B10}). В последнем столбце новой матрицы получаем корни уравнения.

Примеры решения уравнений методом итераций в Excel

Вычисления в книге должны быть настроены следующим образом:

Делается это на вкладке «Формулы» в «Параметрах Excel». Найдем корень уравнения х – х3 + 1 = 0 (а = 1, b = 2) методом итерации с применением циклических ссылок. Формула:

Хn+1 = Xn– F (Xn) / M, n = 0, 1, 2, … .

M – максимальное значение производной по модулю. Чтобы найти М, произведем вычисления:

f’ (1) = -2 * f’ (2) = -11.

Полученное значение меньше 0. Поэтому функция будет с противоположным знаком: f (х) = -х + х3 – 1. М = 11.

В ячейку А3 введем значение: а = 1. Точность – три знака после запятой. Для расчета текущего значения х в соседнюю ячейку (В3) введем формулу: =ЕСЛИ(B3=0;A3;B3-(-B3+СТЕПЕНЬ(B3;3)-1/11)).

В ячейке С3 проконтролируем значение f (x): с помощью формулы =B3-СТЕПЕНЬ(B3;3)+1.

Корень уравнения – 1,179. Введем в ячейку А3 значение 2. Получим тот же результат:

Скачать решения уравнений в Excel

Корень на заданном промежутке один.

Решение системы линейных уравнений методом гаусса-жордана

Здесь вы сможете бесплатно решить систему линейных уравнений методом Гаусса онлайн больших размеров в комплексных числах с очень подробным решением. Наш калькулятор умеет решать онлайн как обычную определенную, так и неопределенную систему линейных уравнений методом Гаусса, которая имеет бесконечное множество решений. В этом случае в ответе вы получите зависимость одних переменных через другие, свободные. Также можно проверить систему уравнений на совместность онлайн, используя решение методом Гаусса.

О методе

При решении системы линейных уравнений онлайн методом Гаусса выполняются следующие шаги.

  1. Записываем расширенную матрицу.
  2. Фактически решение разделяют на прямой и обратный ход метода Гаусса. Прямым ходом метода Гаусса называется приведение матрицы к ступенчатому виду. Обратным ходом метода Гаусса называется приведение матрицы к специальному ступенчатому виду. Но на практике удобнее сразу занулять то, что находится и сверху и снизу рассматриваемого элемента. Наш калькулятор использует именно этот подход.
  3. Важно отметить, что при решении методом Гаусса, наличие в матрице хотя бы одной нулевой строки с НЕнулевой правой частью (столбец свободных членов) говорит о несовместности системы. Решение линейной системы в таком случае не существует.

Чтобы лучше всего понять принцип работы алгоритма Гаусса онлайн введите любой пример, выберите «очень подробное решение» и посмотрите его решение онлайн.

метод Гаусса–Жордана — один из наиболее известных и широко применяемых методов решения систем линейных уравнений. Матричный метод и метод Крамера обладают тем недостатком, что они не дают ответа в том случае, когда detA = 0, а определяют лишь единственное решение при detA неравном 0. Еще одним недостатком является то, что объем математических вычислений в рамках этих методов резко возрастает с ростом числа уравнений. Метод Гаусса практически свободен от этих недостатков.

Алгоритм метода Гаусса

  1. На основании системы линейных уравнений составляем расширенную матрицу системы;
  2. Приводим матрицу к «треугольному» виду;
  3. Определяем ранги основной и расширенной матриц, и на основании этого делаем вывод о совместности системы и количестве допустимых решений;
  4. В случае, если система имеет единственное решение производим обратную подстановку и находим его, если система имеет множество решений: выражаем базисные переменные через переменные которые могут принимать произвольные значения;
Комментарий к шагу 2 Метода Гаусса. Треугольной называют матрицу, в которой все элементы расположенные ниже главной диагонали равны нулю.

Для приведения исходной расширенной матрицы к треугольному виду используем следующие два свойства определителей:

Свойство 1. Определитель не изменит свое значение, если ко всем элементам какой-либо строки (столбца) матрицы прибавить соответствующие элементы параллельной строки (столбца), умноженные на произвольное одно и то же число.

Свойство 2. При перестановке двух любых столбцов или строк матрицы ее определитель меняет знак на противоположный, а абсолютная величина определителя остается неизменной.

На основании этих свойств определителей составим алгоритм преобразования матрицы к треугольному виду:

  1. Рассматриваем строку i(начиная с первой). Если, элемент a i i равен нулю, меняем местами i-ю и i+1-ю строки матрицы. Знак определителя при этом изменится на противоположный. Если a 1 1 отличен от нуля — переходим к следующему шагу;
  2. Для каждой строки j, ниже i-й находим значение коэффициента K j =a j i /a i i ;
  3. Пересчитываем элементы всех строк j, расположенных ниже текущей строки i, с использованием соответствующих коэффициентов по формуле: a j k нов.=a j k -K j *a i k ; После чего, возвращаемся к первому шагу алгоритма и рассматриваем следующую строку, пока не доберемся до строки i=n-1, где n — размерность матрицы A
  4. В полученной треугольной матрице расчитываем произведение всех элементов главной диагонали Пa i i , которое и будет являтся определителем;

Другими словами, суть метода можно сформулировать следующим образом. Нам необходимо сделать нулевыми все элементы матрицы ниже главной диагонали. Сначала мы получаем нули в первом столбце. Для этого мы последовательно вычитаем первую строку, домноженную на нужное нам число (такое, чтоб при вычитании мы получили ноль в первом элементе строки), из всех ниже лежащих строк. Затем проделываем то же самое для второй строки, чтобы получить нули во втором столбце ниже главной диагонали матрицы. И так далее пока не доберемся до предпоследней строки.

4. Метод Жордана — Гаусса.

Схема с выбором главного элемента состоит в том, что требование неравенства нулю диагональных элементов akk, на которые происходит деление в процессе исключения, заменятся более жестким: из всех элементов К-го столба выбрать наибольший по модулю и переставить уравнения так, чтобы этот элемент оказался на месте элемента акк. Выбор главного элемента и связанная с ним перестановка строк необходимы в тех случаях, когда на каком-либо i-ом шаге акк=0 либо же акк очень мало по остальными элементами i- го столбца: при делении на такое «малое» акк будут получаться большие числа с большими абсолютными погрешностями, в результате чего решение может сильно исказиться.

Ниже излагается алгоритм полного исключения неизвестных или метод Жордана – Гаусса. Суть метода состоит в том, что, рассмотрев первое уравнение, в нем неизвестное с коеффициэнтом, отличным от нуля (в дальнейшем разрешающий элемент), и разделив первое уравнение на этот коэффициент, с помощью первого уравнения исключают это неизвестное из всех уравнений, кроме первого. Выбрав во втором уравнении неизвестное с коэффициентом, отличным от нуля, и разделив на него второе уравнение, с помощью второго исключают другие неизвестные из всех уравнений, кроме второго и т.д., т.е. с помощью одного уравнения производят полное исключение одного неизвестного. Процесс продолжается до тех пор, пока не будут использованы все уравнения.

Как известно, системы линейных алгебраических уравнений могут имеет одно решение, множество решений или системы несовместны. При элементарных преобразованиях элементов матрицы системы эти случаи выявляются в следующем:

1. В процессе исключений левая часть I –го уравнения системы обращается в нуль, а правая часть равна некоторому числу, отличному от нуля. т.е. 02+=bc0.

Это означает, что система не имеет решений, так как I – му уравнению не могут удовлетворять никакие значения неизвестных;

2. Левая и правая части I – го уравнения обращаются в нуль. Это означает, что I – ое уравнение является линейной комбинацией остальных, ему удовлетворяет любое найденное решение системы, поэтому оно может быть отброшено. В системе количество неизвестных больше количества уравнений и, следовательно, такая система имеет множество решений;

3. После того как все уравнения использованы для исключения неизвестных получено решение системы.

Таким образом, конечной целью преобразований Жордана-Гаусса является получение из заданной линейной системы

a11x1 + a12x2 + … + a1nxn = b1,n+1

a21x1 + a22x2 + … + a2nxn = b2,n+1

am1x1 + am2x2 + … + amnxn = bm.n+1

Здесь x1, x2, …, xn — неизвестные, которые надо определить. a11, a12, …, amn — коэффициенты системы — и b1, b2, … bm — свободные члены — предполагаются известными. Индексы коэффициентов (aij) системы обозначают номера уравнения (i) и неизвестного (j), при котором стоит этот коэффициент, соответственно.

Система (1) называется однородной, если все её свободные члены равны нулю (b1 = b2 = … = bm = 0), иначе — неоднородной.

Система (1) называется квадратной, если число m уравнений равно числу n неизвестных.

Решение системы (1) — совокупность n чисел c1, c2, …, cn, таких что подстановка каждого ci вместо xi в систему (1) обращает все ее уравнения в тождества.

Система (1) называется совместной, если она имеет хотя бы одно решение, и несовместной, если у нее нет ни одного решения.

Совместная система вида (1) может иметь одно или более решений.

Решения c1(1), c2(1), …, cn(1) и c1(2), c2(2), …, cn(2) совместной системы вида (1) называются различными, если нарушается хотя бы одно из равенств:

c1(1) = c1(2), c2(1) = c2(2), …, cn(1) = cn(2).

Совместная система вида (1) называется определенной, если она имеет единственное решение; если же у нее есть хотя бы два различных решения, то она называется неопределенной. Если уравнений больше, чем неизвестных, она называется переопределённой.

Решим следующую систему уравнений:

Запишем её в виде матрицы 3×4, где последний столбец является свободным членом:

Проведём следующие действия:

· К строке 2 добавим: -4 * Строку 1.

· К строке 3 добавим: -9 * Строку 1.

· К строке 3 добавим: -3 * Строку 2.

· Строку 2 делим на -2

· К строке 1 добавим: -1 * Строку 3.

· К строке 2 добавим: -3/2 * Строку 3.

· К строке 1 добавим: -1 * Строку 2.

В правом столбце получаем решение:

.

В методе Ньютона наблюдается ускорение сходимости процесса приближений. 5. Метод касательных (метод Ньютона) Метод касательных, связанный с именем И. Ньютона, является одним из наиболее эффективных численных методов решения уравнений. Идея метода очень проста. Возьмём производную точку x0 и запишем в ней уравнение касательной к графику функции f(x): y=f(x0)+ f ¢(x) (x-x0) (1.5) Графики…

Решения от численных методов расчёта. Для определения корней уравнения не требуется знания теорий групп Абеля, Галуа, Ли и пр. и применения специальной математической терминологии: колец, полей, идеалов, изоморфизмов и т.д. Для решения алгебраического уравнения n — ой степени нужно только умение решать квадратные уравнения и извлекать корни из комплексного числа. Корни могут быть определены с…



Математики тригонометрической подстановки и проверка эффективности разработанной методики преподавания. Этапы работы: 1. Разработка факультативного курса на тему: «Применение тригонометрической подстановки для решения алгебраических задач» с учащимися классов с углубленным изучением математики. 2. Проведение разработанного факультативного курса. 3. Проведение диагностирующей контрольной…

… «проявляется» лишь в процессе преобразований. Очевидность и «завуалированность» новой переменной мы рассмотрим на конкретных примерах во второй главе данной работы. 2. Возможности применения метода замены неизвестного при решении алгебраических уравнений В этой главе выявим возможности применения метода замены неизвестного при решении алгебраических уравнений в стандартных и нестандартных…

Однажды немецкий математик Вильгельм Йордан (мы неверно транскрибируем с немецкого Jordan как Жордан) сел решать очередную систему уравнений. Он любил этим заниматься и в свободное время совершенствовал свои навыки. Но вот настал момент, когда ему наскучили все методы решения и метод Гаусса в том числе…

Предположим, дана система с тремя уравнениями, тремя неизвестными и записана её расширенная матрица . В наиболее распространенном случае получаются стандартные ступеньки , и так каждый день…. Одно и то же – как беспросветный ноябрьский дождь.

На некоторое время развевает тоску другой способ приведения матрицы к ступенчатому виду: , причём он совершенно равноценен и может быть неудобен только по причине субъективного восприятия. Но всё рано или поздно приедается…. И подумал тогда Жо рдан – а зачем вообще мучиться с обратным ходом гауссовского алгоритма? Не проще ли сразу получить ответ с помощью дополнительных элементарных преобразований?

…да, такое бывает только по любви =)

Для освоения данного урока «чайникам» придётся пойти путём Жо рдана и прокачать элементарные преобразования хотя бы среднего уровня, прорешав, минимум, 15-20 соответствующих заданий. Поэтому если вы смутно понимаете, о чём идёт разговор и/или у вас возникнет недопонимание чего-либо по ходу занятия, то рекомендую ознакомиться с темой в следующем порядке:

Ну, и совсем замечательно, если отработано понижение порядка определителя .

Как все поняли, метод Гаусса-Жордана представляет собой модификацию метода Гаусса и с реализацией основной, уже озвученной выше идеи, мы встретимся на ближайших экранах. Кроме того, в число немногочисленных примеров данной статьи вошло важнейшее приложение – нахождение обратной матрицы с помощью элементарных преобразований .

Не мудрствуя лукаво:

Пример 1

Решить систему методом Гаусса-Жордана

Решение : это первое задание урока Метод Гаусса для чайников , где мы 5 раз трансформировали расширенную матрицу системы и привели её к ступенчатому виду:

Теперь вместо обратного хода в игру вступают дополнительные элементарные преобразования. Сначала нам необходимо получить нули на этих местах: ,
а потом ещё один ноль вот здесь: .

Идеальный с точки зрения простоты случай:

(6) Ко второй строке прибавили третью строку. К первой строке прибавили третью строку.

(7) К первой строке прибавили вторую строку, умноженную на –2.

Не могу удержаться от иллюстрации итоговой системы:

Ответ :

Предостерегаю читателей от шапкозакидательского настроения – это был простейший демонстрационный пример. Для метода Гаусса-Жордана характерны свои специфические приёмы и не самые удобные вычисления, поэтому, пожалуйста, настройтесь на серьёзную работу.

Не хочу показаться категоричным или придирчивым, но в подавляющем большинстве источников информации, которые я видел, типовые задачи рассмотрены крайне плохо – нужно обладать семью пядями во лбу и потратить массу времени/нервов на тяжёлое неуклюжее решение с дробями. За годы практики мне удалось отшлифовать, не скажу, что самую лучшую, но рациональную и достаточно лёгкую методику, которая доступна всем, кто владеет арифметическими действиями:

Пример 2

Решить систему линейных уравнений методом Гаусса-Жордана.

Решение : первая часть задания хорошо знакома:

(1) Ко второй строке прибавили первую строку, умноженную на –1. К третьей строке прибавили первую строку, умноженную на 3. К четвертой строке прибавили первую строку, умноженную на –5.

(2) Вторую строку разделили на 2, третью строку разделили на 11, четвёртую строку разделили на 3.

(3) Вторая и третья строки пропорциональны, 3-ю строку удалили. К четвёртой строке прибавили вторую строку, умноженную на –7

(4) Третью строку разделили на 2.

Очевидно, что система имеет бесконечно много решений, и наша задача – привести её расширенную матрицу к виду .

Как действовать дальше? Прежде всего, следует отметить, что мы лишились вкусного элементарного преобразования – перестановки строк. Точнее говоря, переставить-то их можно, но в этом нет смысла (просто выполним лишние действия). И далее целесообразно придерживаться следующего шаблона:

Находим наименьшее общее кратное чисел третьего столбца (1, –1 и 3), т.е. – наименьшее число, которое бы делилось без остатка и на 1, и на –1 и на 3. В данном случае, это, конечно же, «тройка». Теперь в третьем столбце нам нужно получить одинаковые по модулю числа , и этими соображениями обусловлено 5-е преобразование матрицы:

(5) Первую строку умножаем на –3, вторую строку умножаем на 3. Вообще говоря, первую строку можно было умножить тоже на 3, но это было бы менее удобно для следующего действия. К хорошему привыкаешь быстро:


(6) Ко второй строке прибавили третью строку. К первой строке прибавили третью строку.

(7) Во втором столбце два ненулевых значения (24 и 6) и нам снова нужно получить одинаковые по модулю числа . В данном случае всё сложилось довольно удачно – наименьшее кратное 24, и эффективнее всего умножить вторую строку на –4.

(8) К первой строке прибавили вторую.

(9) Заключительный штрих: первую строку разделили на –3, вторую строку разделили на –24 и третью строку разделили на 3. Это действие выполняется В ПОСЛЕДНЮЮ ОЧЕРЕДЬ! Никаких преждевременных дробей!

В результате элементарных преобразований получена эквивалентная исходной система:

Элементарно выражаем базисные переменные через свободную:

и записываем:

Ответ : общее решение:

В подобных примерах применение рассмотренного алгоритма чаще всего оправдано, поскольку обратный ход метода Гаусса обычно требует трудоёмких и неприятных вычислений с дробями.

И, разумеется, крайне желательна проверка, которая выполняется по обычной схеме, рассмотренной на уроке Несовместные системы и системы с общим решением .

Для самостоятельного решения:

Пример 3

Найти базисное решение с помощью элементарных преобразований

Такая формулировка задачи предполагает использование метода Гаусса-Жордана, и в образце решения матрица приводится к стандартному виду с базисными переменными . Однако всегда держите на заметке, что в качестве базисных можно выбрать и другие переменные . Так, например, если в первом столбце громоздкие числа, то вполне допустимо привести матрицу к виду (базисные переменные ), или к виду (базисные переменные ), или даже к виду с базисными переменными . Существуют и другие варианты.

Но всё-таки это крайние случаи – не стОит лишний раз шокировать преподавателей своими знаниями, техникой решения и уж тем более не надо выдавать экзотических жордановсих результатов вроде . Впрочем, бывает трудно удержаться от нетипового базиса, когда в исходной матрице, скажем, в 4-м столбце есть два готовых нуля.

Примечание : термин «базис» имеет алгебраический смысл и понятие геометрического базиса здесь не при чём!

Если в расширенной матрице данных размеров вдруг обнаруживается пара линейно зависимых строк, то её следует попытаться привести к привычному виду с базисными переменными . Образец такого решения есть в Примере №7 статьи об однородных системах линейных уравнений , причём там выбран другой базис .

Продолжаем совершенствовать свои навыки на следующей прикладной задаче:

Как найти обратную матрицу методом Гаусса?

Обычно условие формулируют сокращённо, но, по существу, здесь также работает алгоритм Гаусса-Жордана. Более простой метод нахождения обратной матрицы для квадратной матрицы мы давным-давно рассмотрели на соответствующем уроке, и суровой поздней осенью тёртые студенты осваивают мастерский способ решения.

Краткое содержание предстоящих действий таково: сначала следует записать квадратную матрицу в тандеме с единичной матрицей: . Затем с помощью элементарных преобразований необходимо получить единичную матрицу слева, при этом (не вдаваясь в теоретические подробности) справа нарисуется обратная матрица. Схематически решение выглядит следующим образом:

(Понятно, что обратная матрица должна существовать)

Демо-пример 4

Найдём обратную матрицу для матрицы с помощью элементарных преобразований. Для этого запишем её в одной упряжке с единичной матрицей, и понеслась «двойка скакунов»:

(1) Ко второй строке прибавили первую строку, умноженную на –3.

(2) К первой строке прибавили вторую строку.

(3) Вторую строку разделили на –2.

Ответ :

Сверьтесь с ответом первого примера урока Как найти обратную матрицу?

Но то была очередная заманивающая задачка – в действительности решение гораздо более длительно и кропотливо. Как правило, вам будет предложена матрица «три на три»:

Пример 5


Решение : присоединяем единичную матрицу и начинаем выполнять преобразования, придерживаясь алгоритма «обычного» метода Гаусса :

(1) Первую и третью строки поменяли местами. На первый взгляд, перестановка строк кажется нелегальной, но на самом деле переставлять их можно – ведь по итогу слева нам нужно получить единичную матрицу, а справа же «принудительно» получится именно матрица (вне зависимости от того будем ли мы переставлять строки в ходе решения или нет) . Обратите внимание, что здесь вместо перестановки можно организовать «шестёрки» в 1-м столбце (наименьшее общее кратное (НОК) чисел 3, 2 и 1) . Решение через НОК особенно удобно, когда в первом столбце отсутствуют «единицы».

(2) Ко 2-й и 3-й строкам прибавили 1-ю строку, умноженную на –2 и –3 соответственно.

(3) К 3-й строке прибавили 2-ю строку, умноженную на –1

Вторая часть решения проводится по уже известной из предыдущего параграфа схеме: перестановки строк становятся бессмысленными, и мы находим наименьшее общее кратное чисел третьего столбца (1, –5, 4): 20. Существует строгий алгоритм нахождения НОК, но здесь обычно хватает подбора. Ничего страшного, если взять бОльшее число, которое делится и на 1, и на –5, и на 4, например, число 40. Отличие будет в более громоздких вычислениях.

К слову о вычислениях. Для решения задачи совсем не зазорно вооружиться микрокалькулятором – числа здесь фигурируют немалые, и будет очень обидно допустить вычислительную ошибку.

(4) Третью строку умножаем на 5, вторую строку на 4, первую строку на «минус двадцать»:

(5) К 1-й и 2-й строкам прибавили третью строку.

(6) Первую и третью строки разделили на 5, вторую строку умножили на –1.

(7) Наименьшее общее кратное ненулевых чисел второго столбца (–20 и 44) равно 220. Первую строку умножаем на 11, вторую строку – на 5.

(8) К первой строке прибавили вторую строку.

(9) Первую строку умножили на –1, вторую строку разделили «обратно» на 5.

(10) Теперь на главной диагонали левой матрицы целесообразно получить наименьшее общее кратное чисел диагонали (44, 44 и 4). Совершенно понятно, что это число 44. Третью строку умножаем на 11.

(11) Каждую строку делим на 44. Данное действие выполняется в последнюю очередь!

Таким образом, обратная матрица:

Внесение и вынесение -й, в принципе, лишние действия, но того требует протокол оформления задачи.

Ответ :

Проверка выполняется по обычной схеме, рассмотренной на уроке об обратной матрице .

Продвинутые люди могут несколько сократить решение, но должен предупредить, спешка тут чревата ПОВЫШЕННЫМ риском допустить ошибку.

Аналогичное задание для самостоятельного решения:

Пример 6

Найти обратную матрицу методом Гаусса-Жордана.

Примерный образец оформления задачи внизу страницы. И ради того, чтобы вы «не проехали мимо с песнями» я оформил решение в уже упомянутом стиле – исключительно через НОК столбцов без единой перестановки строк и дополнительных искусственных преобразований. По моему мнению, эта схема – если и не самая, то одна из самых надёжных .

Иногда бывает удобно более короткое «модернистское» решение, которое заключается в следующем: на первом шаге всё как обычно: .

На втором шаге накатанным приёмом (через НОК чисел 2-го столбца) организуются сразу два нуля во втором столбце: . Перед данным действием особенно трудно устоять, если во 2-м столбце нарисовались одинаковые по модулю числа, например, те же банальные «единицы».

И, наконец, на третьем шаге точно так же получаем нужные нули в третьем столбце: .

Что касается размерности, то в большинстве случаев приходится разруливать матрицу «три на три». Однако время от времени встречается лайт-версия задачи с матрицей «два на два» и хард… – специально для всех читателей сайт:

Пример 7

Найти обратную матрицу с помощью элементарных преобразований

Это задание из моей собственной физматовской контрольной работы по алгебре, …эх, где мой первый курс =) Пятнадцать лет назад (листочек на удивление ещё не пожелтел) , я уложился в 8 шагов, а сейчас – всего лишь в 6! Матрица, кстати, весьма творческая – на первом же шаге просматривается несколько заманчивых путей решения. Моя поздняя версия внизу страницы.

И заключительный совет – после таких примеров очень полезна гимнастика для глаз и какая-нибудь хорошая музыка для релаксации =)

Желаю успехов!

Решения и ответы:

Пример 3: Решение : запишем расширенную матрицу системы и с помощью элементарных преобразований получим базисное решение:


(1) Первую и вторую строки поменяли местами.

(2) Ко второй строке прибавили первую строку, умноженную на –2. К третьей строке прибавили первую строку, умноженную на 5.
(3) Третью строку разделили на 3.
(4) К третьей строке прибавили вторую строку, умноженную на 2.
(5) Третью строку разделили на 7.
(6) Наименьшее кратное чисел 3-го столбца (–3, 5, 1) равно 15. Первую строку умножили на 5, вторую строку умножили на –3, третью строку умножили на 15.
(7) К первой строке прибавили 3-ю строку. Ко второй строке прибавили 3-ю строку.
(8) Первую строку разделили на 5, вторую строку разделили на –3, третью строку разделили на 15.
(9) Наименьшее кратное ненулевых чисел 2-го столбца (–2 и 1) равно: 2. Вторую строку умножили на 2
(10) К первой строке прибавили вторую строку.
(11) Вторую строку разделили на 2.
Выразим базисные переменные через свободные переменные :

Ответ : общее решение:

Пример 6: Решение : обратную матрицу найдём с помощью элементарных преобразований:


(1) Первую строку умножили на –15, вторую строку умножили на 3, третью строку умножили на 5.

(2) Ко 2-й и 3-й строкам прибавили первую строку.
(3) Первую строку разделили на –15, вторую строку разделили на –3, третью строку разделили на –5.
(4) Вторую строку умножили на 7, третью строку умножили на –9.
(5) К третьей строке прибавили вторую строку.


(6) Вторую строку разделили на 7.

(7) Первую строку умножили на 27, вторую строку умножили на 6, третью строку умножили на –4.
(8) К первой и второй строкам прибавили третью строку.
(9) Третью строку разделили на –4. К первой строке прибавили вторую строку, умноженную на –1.
(10) Вторую строку разделили на 2.
(11) Каждую строку разделили на 27.
В результате:
Ответ :

Пример 7: Решение : найдём обратную матрицу методом Гаусса-Жордана:
(1) К 1-й и 4-й строкам прибавили 3-ю строку.
(2) Первую и четвёртую строки поменяли местами.
(3) Ко 2-й строке прибавили 1-ю строку. К 3-й строке прибавили 1-ю строку, умноженную на 2:


(4) К 3-й строке прибавили 2-ю строку, умноженную на –2. К 4-й строке прибавили 2-ю строку.
(5) К 1-й и 3-й строкам прибавили 4-ю строку, умноженную на –1.
(6) Вторую строку умножили на –1, третью строку разделили на –2.
Ответ :

Записывается в виде расширенной матрицы, т.е. в столбец свободных членов помещается в одну матрицу с коэффициентами неизвестных. Аалгоритм заключается в приведении исходной матрицы, характеризующей систему линейных уравнений, к единичной путем эквивалентных преобразований (домножения строки матрицы на константу и сложения с другой строкой матрицы). В качестве константы используется 1/a[i][i] , т.е. число, обратное по отношению к элементу диагонали. Естественно, в ряде случаев возникают проблемы, связанные с делением на ноль, которые решаются перестановкой строк и столбцов:

Весь алгоритм можно представить 10 пунктами:

    В качестве опорной выбираем первую строку матрицы.

    Если элемент опорной строки, индекс которого равен номеру опорной строки, равен нулю, то меняем всю опорную строку на первую попавшуюся строку снизу, в столбце которого нет нуля.

    Все элементы опорной строки делим на первый слева ненулевой элемент этой строки.

    Из оставшихся снизу строк вычитают опорную строку, умноженную на элемент, индекс которого равен номеру опорной строки.

    В качестве опорной строки выбираем следующую строку.

    Повторяем действия 2 – 5 пока номер опорной строки не превысит число строк.

    В качестве опорной выбираем последнюю строку.

    Вычитаем из каждой строки выше опорную строку, умноженную на элемент этой строки с индексом равным номеру опорной строки.

    В качестве опорной строки выбираем строку выше.

    Повторяем 8 – 9 пока номер опорной строки не станет меньше номера первой строки.

Пусть имеется система уравнений:

Запишем расширенную матрицу системы:

и выполним элементарные преобразования ее строк.

Для этого умножим первую строку на 1 и вычитаем из второй строки; затем умножим первую строку на 2 и вычтем из третьей строки.

В результате мы исключим переменную x 1 из всех уравнений, кроме первого. Получим:

Теперь вычтем из строки 3 строку 2, умноженную на 3:

Теперь вычитаем из 1 строки сначала 3 строку, а затем 2 строку:

После преобразований получаем систему уравнений:

Из этого следует, что система уравнений имеет следующее решение:

x1 = 1, x2 = 3 , x3 = -1

    В качестве примера решим систему уравнений, представленную в виде матрицы (Таблица 1), методом Гаусса – Жордана.

Делим первую строку на 3 (элемент первой строки, расположенный на главной диагонали), получим:

Умножаем первую строку на 1 и вычитаем из второй строки. Умножаем первую строку на 6 и вычитаем из третьей строки. Получим:

В первом столбце все элементы кроме диагонального равны нулю, займемся вторым столбцом, для этого выберем вторую строку в качестве опорной. Вторая Делим ее на 17/3:

Умножаем строку 2 на -6 и вычитаем из третьей строки:

Теперь третья строка – опорная, делим ее на -33/17:

Умножаем опорную строку на 3/17 и вычитаем ее из второй. Умножаем третью строку на 1 и вычитаем ее из первой

Получена треугольная матрица, начинается обратный ход алгоритма (во время которого получим единичную матрицу). Вторая строка становится опорной. Умножаем третью строку на 4/3 и вычитаем ее из первой:

Последний столбец матрицы – решение системы уравнений.


Калькулятор метода исключения Гаусса

— Онлайн-программа для сокращения строк

Поиск инструмента

Исключение по Гауссу

Инструмент для применения метода исключения Гаусса и получения формы сокращенного эшелона строки с шагами, деталями, обратной матрицей и векторным решением.

Результаты

Исключение Гаусса — dCode

Тег (и): Матрица, символьное вычисление

Поделиться

dCode и другие

dCode является бесплатным, а его инструменты являются ценным подспорьем в играх, математике, геокешинге, головоломках и задачах, которые нужно решать каждый день!
Предложение? обратная связь? Жук ? идея ? Запись в dCode !

Калькулятор исключения по Гауссу

Преобразователь системы уравнений в матрицу

Ответы на вопросы (FAQ)

Что такое метод исключения Гаусса?

Алгоритм исключения Гаусса (также называемый методом Гаусса-Жордана или методом поворота) позволяет находить решения системы линейных уравнений и определять обратную матрицу.

Алгоритм работает со строками матрицы путем обмена или умножения строк между ними (с точностью до множителя).

На каждом шаге алгоритм стремится ввести в матрицу на элементах за пределами диагонали нулевые значения.

Как вычислить решения системы линейных уравнений с Гауссом?

Первым шагом из системы линейных уравнений является преобразование уравнений в матрицу.

Пример: $$ \ left \ {\ begin {array} {} x & — & y & + & 2z & = & 5 \\ 3x & + & 2y & + & z & = & 10 \\ 2x & — & 3y & — & 2z & = & — 10 \\\ end {массив} \ право.$$ можно записать в форме умножения «> матричного умножения: $$ \ left (\ begin {array} {ccc} 1 & -1 & 2 \\ 3 & 2 & 1 \\ 2 & -3 & 2 \ end { array} \ right). \ left (\ begin {array} {c} x \\ y \\ z \ end {array} \ right) = \ left (\ begin {array} {c} 5 \\ 10 \\ -10 \ end {array} \ right) $$, который соответствует (расширенной) матрице $$ \ left (\ begin {array} {ccc | c} 1 & -1 & 2 & 5 \\ 3 & 2 & 1 & 10 \\ 2 & -3 & 2 & -10 \ end {array} \ right) $$

Затем для каждого элемента за пределами ненулевой диагонали выполните соответствующие вычисления, добавляя или вычитая другие строки, чтобы элемент стал 0.

Пример: Отнимите 3 раза (строка 1) до (строка 2), например, элемент в строке 2, столбец 1 станет 0: $$ \ left (\ begin {array} {ccc | c} 1 & -1 & 2 & 5 \\ 0 & 5 & -5 & -5 \\ 2 & -3 & -2 & -10 \ end {array} \ right) $$
Вычтите 2 раза (строка 1) до (строка 3) например, элемент в строке 3, столбец 1 становится 0: $$ \ left (\ begin {array} {ccc | c} 1 & -1 & 2 & 5 \\ 0 & 5 & -5 & -5 \\ 0 & -1 & -6 & -20 \ end {array} \ right) $$
Вычтите 1/5 раз (строка 2) из ​​(строка 3), например, элемент в строке 3, столбец 2 станет 0: $$ \ слева (\ begin {array} {ccc | c} 1 & -1 & 2 & 5 \\ 0 & 5 & -5 & -5 \\ 0 & 0 & -7 & -21 \ end {array} \ right) $$
Вычтите 1/5 раз (строка 2) из ​​(строка 1), например, элемент в строке 1, столбец 2 станет 0: $$ \ left (\ begin {array} {ccc | c} 1 & 0 & 1 & 4 \\ 0 & 5 & -5 & -5 \\ 0 & 0 & -7 & -21 \ end {array} \ right) $$
Отнимите 1/7 раз (строка 3) до (строка 1), например как элемент в строке 1, столбец 3 становится 0: $$ \ left (\ begin {array} {ccc | c} 1 & 0 & 0 & 1 \\ 0 & 5 & -5 & -5 \\ 0 & 0 & -7 & -21 \ end {array} \ right) $$
Вычтите 5/7 раз (строка 3) из (строка 2), например, элемент в строке 2, столбец 3 станет 0: $$ \ left (\ begin {array} {ccc | c} 1 & 0 & 0 & 1 \\ 0 & 5 & 0 & 10 \\ 0 & 0 & -7 & -21 \ end {array} \ right) $$

Упростите каждую строку, разделив значение по диагонали.

Пример: $$ \ left (\ begin {array} {ccc | c} 1 & 0 & 0 & 1 \\ 0 & 1 & 0 & 2 \\ 0 & 0 & 1 & 3 \ end {array } \ right) $$

Результирующий вектор — это последний столбец.

Пример: $ {1,2,3} $, что соответствует $ {x, y, z} $, поэтому $ x = 1, y = 2, z = 3 $

Задайте новый вопрос

Исходный код

dCode сохраняет за собой право собственности на исходный код онлайн-инструмента «Исключение Гаусса». За исключением явной лицензии с открытым исходным кодом (обозначенной CC / Creative Commons / бесплатно), любой алгоритм, апплет или фрагмент алгоритма исключения Гаусса (конвертер, решатель, шифрование / дешифрование, кодирование / декодирование, шифрование / дешифрование, переводчик) или любой алгоритм исключения Гаусса ‘функция (вычислить, преобразовать, решить, расшифровать / зашифровать, расшифровать / зашифровать, декодировать / закодировать, перевести), написанная на любом информатическом языке (Python, Java, PHP, C #, Javascript, Matlab и т. д.)), и никакая загрузка данных, скрипт, копипаст или доступ к API для «Исключения Гаусса» не будут бесплатными, то же самое для автономного использования на ПК, планшете, iPhone или Android! dCode распространяется бесплатно и онлайн.

Нужна помощь?

Пожалуйста, посетите наше сообщество dCode Discord для запросов о помощи!
NB: для зашифрованных сообщений проверьте наш автоматический идентификатор шифра!

Вопросы / комментарии

Сводка

Похожие страницы

Поддержка

Форум / Справка

Ключевые слова

исключение, точка поворота, гаусс, иордания, матрица, система, уравнение

Ссылки


Источник: https: // www.dcode.fr/gaussian-elimination

© 2021 dCode — Идеальный «инструментарий» для решения любых игр / загадок / геокэшинга / CTF.

Калькулятор исключения Гаусса

Как найти неизвестные переменные в уравнениях методом исключения Гаусса?

Исключение Гаусса или сокращение строки , это алгоритм для решения системы линейных уравнений. Этот метод также называется исключением Гаусса-Жордана. Он представлен последовательностью операций, выполняемых над матрицей.Метод назван в честь Карла Фридриха Гаусса (1777-1855), хотя был известен китайским математикам. Метод решения системы линейных уравнений методом исключения Гаусса аналогичен методу решения матриц. Например, существует связь между системой трех линейных уравнений и ее матрицей коэффициентов. $$ \ begin {align} & a_1x + b_1y + c_1z = {d_1} \\ & a_2x + b_2y + c_2z = {d_2} \\ & a_3x + b_3y + c_3z = {d_3} \\ \ end {align} \ quad \ longmapsto \ left ( \ begin {array} {ccc} {a_1} & b_1 & c_1 \\ {a_2} & b_2 & c_2 \\ {a_3} & b_3 & c_3 \\ \ end {массив} \ right) $$ Есть три типа операций с элементарными строками:

  • Замена двух рядов;
  • Умножение строки на ненулевое число;
  • Добавление числа, кратного одной строке, к другой строке.
Метод исключения Гаусса состоит из двух частей. Первая часть сводит данную систему к \ underline {форме эшелона строк}. Из формы эшелона строк мы можем сделать вывод, что у системы нет решений, единственное решение или бесконечно много решений. Во второй части используются строковые операции до тех пор, пока не будет найдено решение. Форма рядного эшелона удовлетворяет следующим свойствам:
  • Старший коэффициент каждой строки должен составлять 1 доллар;
  • Все элементы в столбце ниже $ 1 $ должны быть $ 0 $;
  • Все строки, содержащие нули, находятся внизу матрицы.
Например, следующие матрицы представлены в виде эшелона строк $$ \ left ( \ begin {array} {cc} 1 и 5 \\ 0 и 1 \\ \ end {массив} \ вправо), \ квад \ влево ( \ begin {array} {cccc} 1 и 1 и 0 и 5 \\ 0 и 1, 3 и 4 \\ 0 & 0 & 1 & 2 \\ \ end {массив} \ справа), \ quad \ left ( \ begin {array} {cccc} 1 и 2 и 3 и 4 \\ 0 и 1, 3 и 4 \\ 0 & 0 & 1 & 2 \\ 0 & 0 & 0 & 0 \\ \ end {массив} \ right) $$ Матрица находится в виде сокращенного эшелона строк , если, кроме того, в каждом столбце, содержащем ведущий коэффициент, все другие записи в этом столбце равны нулю.Например, матрицы, показанные ниже, являются примерами матриц в сокращенной форме эшелона строк. $$ \ left ( \ begin {array} {cc} 1 & 0 \\ 0 и 1 \\ \ end {массив} \ справа), \ quad \ left ( \ begin {array} {cccc} 1 & 0 & 0 & 7 \\ 0 & 1 & 0 & -2 \\ 0 & 0 & 1 & 2 \\ \ end {массив} \ справа), \ quad \ left ( \ begin {array} {cccc} 1 & 0 & 0 & 2 \\ 0 & 1 & 0 & -2 \\ 0 & 0 & 1 & 2 \\ 0 & 0 & 0 & 0 \\ \ end {массив} \ right) $$ Расширенная матрица — это матрица, полученная путем добавления столбцов двух заданных матриц.В случае решения системы нам необходимо увеличить матрицу коэффициентов и постоянную матрицу. Вертикальная линия указывает разделение между матрицей коэффициентов и постоянной матрицей. Итак, для системы трех уравнений $$ \ begin {align} & a_1x + b_1y + c_1z = {d_1} \\ & a_2x + b_2y + c_2z = {d_2} \\ & a_3x + b_3y + c_3z = {d_3} \\ \ end {align} $$ расширенная матрица $$ \ left ( \ begin {array} {ccc | c} a_1 & b_1 & c_1 & d_1 \\ a_2 & b_2 & c_2 & d_2 \\ a_3 & b_3 & c_3 & d_3 \\ \ end {массив} \ right) $$ Количество решений системы зависит только от ранга матрицы, представляющей систему, и ранга соответствующей расширенной матрицы.На основании теоремы Кронекера-Капелли любая система из трех линейных уравнений не имеет решений, если ранг расширенной матрицы больше ранга матрицы коэффициентов. Если ранги этих двух матриц равны, система должна иметь хотя бы одно решение. Решение уникально тогда и только тогда, когда ранг равен количеству переменных, в данном случае, если ранг равен 3 $. Например, решим решение системы методом исключения Гаусса $$ \ begin {align} & 4x + 5y + 3z = {10} \\ & 3x + 6y + 7z = {8} \\ & 2x + 3y + 0z = {8} \\ \ end {align} $$ Коэффициенты и постоянные члены системы дают матрицы $$ \ left ( \ begin {array} {ccc} 4 и 5 и 3 \\ 3 и 6 и 7 \\ 2 и 3 и 0 \\ \ end {массив} \ справа), \ quad \ left ( \ begin {array} {c} 10 \\ 8 \\ 8 \\ \ end {массив} \ right) $$ Расширенная матрица $$ \ left ( \ begin {array} {ccc | c} 4 и 5 и 3 и 10 \\ 3 и 6 и 7 и 8 \\ 2 и 3 и 0 и 8 \\ \ end {массив} \ right) $$ Чтобы решить систему, приведите расширенную матрицу к сокращенной форме эшелона строк следующим образом.
  • Разделите строку $ 1 $ на $ 4 $ ($ R_1 = \ frac {R_1} 4) $, чтобы получить $$ \ left ( \ begin {array} {ccc | c} 1 & \ frac 54 & \ frac 34 & \ frac {5} 2 \\ 3 и 6 и 7 и 8 \\ 2 и 3 и 0 и 8 \\ \ end {массив} \ right) $$
  • Вычтите строку $ 1 $, умноженную на $ 3 $, из строки $ 2 $ ($ R_2 = R_2-3R_1 $), чтобы получить $$ \ left ( \ begin {array} {ccc | c} 1 & \ frac 54 & \ frac 34 & \ frac {5} 2 \\ 0 & \ frac 94 & \ frac {19} 4 & \ frac 12 \\ 2 и 3 и 0 и 8 \\ \ end {массив} \ справа) $$
  • Вычтите строку $ 1 $, умноженную на $ 2 $, из строки $ 3 $ ($ R_3 = R_3-2R_1 $), чтобы получить $$ \ left ( \ begin {array} {ccc | c} 1 & \ frac 54 & \ frac 34 & \ frac {5} 2 \\ 0 & \ frac 94 & \ frac {19} 4 & \ frac 12 \\ 0 & \ frac12 & — \ frac 32 & 3 \\ \ end {массив} \ справа) $$
  • Умножьте строку $ 2 $ на $ \ frac 49 $ ($ R_2 = \ frac49 R_2 $), чтобы получить $$ \ left ( \ begin {array} {ccc | c} 1 & \ frac 54 & \ frac 34 & \ frac {5} 2 \\ 0 & 1 & \ frac {19} 9 & \ frac 29 \\\ 0 & \ frac12 & — \ frac 32 & 3 \\ \ end {массив} \ справа) $$
  • Вычтите строку $ 2 $, умноженную на $ \ frac 54 $, из строки $ 1 $ ($ R_1 = R_1- \ frac54 R_2 $), чтобы получить $$ \ left ( \ begin {array} {ccc | c} 1 & 0 & — \ frac {17} 9 & \ frac {20} 9 \\ 0 & 1 & \ frac {19} 9 & \ frac 29 \\ 0 & \ frac12 & — \ frac 32 & 3 \\ \ end {массив} \ справа) $$
  • Вычтите строку $ 2 $, умноженную на $ \ frac 12 $, из строки $ 3 $ ($ R_3 = R_3- \ frac12R_2 $), чтобы получить $$ \ left ( \ begin {array} {ccc | c} 1 & 0 & — \ frac {17} 9 & \ frac {20} 9 \\ 0 & 1 & \ frac {19} 9 & \ frac 29 \\ 0 & 0 & — \ frac {23} 9 & \ frac {26} 9 \\ \ end {массив} \ right) $$
  • Умножьте строку $ 3 $ на $ — \ frac9 {23} $ ($ R_3 = — \ frac9 {23} R_3 $), чтобы получить $$ \ left ( \ begin {array} {ccc | c} 1 & 0 & — \ frac {17} 9 & \ frac {20} 9 \\ 0 & 1 & \ frac {19} 9 & \ frac 29 \\ 0 & 0 & 1 & — \ frac {26} {23} \\ \ end {массив} \ справа) $$
  • Добавьте строку $ 3 $, умноженную на $ \ frac {17} 9 $, в строку $ 1 $ ($ R_1 = R_1 + \ frac {17} 9R_3 $), чтобы получить $$ \ left ( \ begin {array} {ccc | c} 1 & 0 & 0 & \ frac2 {23} \\ 0 & 1 & \ frac {19} 9 & \ frac 29 \\ 0 & 0 & 1 & — \ frac {26} {23} \\ \ end {массив} \ справа) $$
  • Вычтите строку $ 3 $, умноженную на $ \ frac {19} 9 $, из строки $ 2 $ ($ R_2 = R_2- \ frac {19} 9R_3 $), чтобы получить $$ \ left ( \ begin {array} {ccc | c} 1 & 0 & 0 & \ frac2 {23} \\ 0 & 1 & 0 & \ frac {60} {23} \\ 0 & 0 & 1 & — \ frac {26} {23} \\ \ end {массив} \ right) $$ Итак, решение системы: $ (x, y, z) = (\ frac {2} {23}, \ frac {60} {23}, — \ frac {26} {23}) $.
Работа исключения Гаусса с шагами показывает полное пошаговое вычисление для поиска решения линейной системы трех уравнений с использованием метода исключения Гаусса. Для любой другой системы просто введите двенадцать действительных чисел в качестве коэффициентов линейных уравнений и нажмите кнопку «Создать работу». Учащиеся начальной школы используют этот Калькулятор исключения Гаусса для создания работы, проверки результатов решения систем линейных уравнений, выведенных вручную, или для эффективного выполнения домашних заданий.Во многих приложениях необходимо вычислить исключение матрицы, где этот онлайн-калькулятор исключения матрицы Гаусса может помочь легко упростить вычисления для соответствующих входных данных.

Онлайн калькулятор: Метод исключения Гаусса

Система линейных уравнений:

может быть решена методом исключения Гаусса с помощью калькулятора.

В методе исключения Гаусса система линейных уравнений представлена ​​как расширенная матрица, то есть матрица, содержащая коэффициенты уравнения и постоянные члены с размерами [n: n + 1]:

Исключение по Гауссу
8 3 4 5 31 14 4 33 23 17 15 4 23 7 22 4 11 17 1 51

Матрица системы линейных уравнений

Точность вычислений

Цифры после десятичной точки: 2

Файл очень большой.Во время загрузки и создания может произойти замедление работы браузера.

Скачать закрыть

content_copy Ссылка сохранить Сохранить расширение Виджет

Исключение по Гауссу

Метод назван в честь Карла Фридриха Гаусса, гениального немецкого математика 19 века. Сам Гаусс не изобрел этот метод. Метод сокращения строк был известен древним китайским математикам; он был описан в «Девяти главах математического искусства», китайской книге по математике, изданной во II веке.

Ликвидация вперед

Первым шагом исключения Гаусса является получение матрицы строковой формы. Левая нижняя часть этой матрицы содержит только нули, и все нулевые строки находятся ниже ненулевых строк:

Матрица приводится к этой форме с помощью элементарных операций со строками: поменять местами две строки, умножить строку на константу, добавить к одной строке скалярное число, кратное другой.
Наш калькулятор получает форму эшелона путем последовательного вычитания верхних строк, умножения на нижние строки, умножения на, где i — ведущая строка коэффициентов (ведущая строка).
Важно, чтобы старший коэффициент отличался от нуля. Если он становится равным нулю, строка заменяется на более низкую с ненулевым коэффициентом в той же позиции.

Обратная замена

На этом этапе операции с элементарными строками продолжаются до тех пор, пока не будет найдено решение. Наконец, он преобразует матрицу в сокращенный ряд строк:
,

Решатель уравнений: Wolfram | Alpha

О решении уравнений

Значение называется корнем полинома if.

Наибольший показатель степени появления называется степенью. Если имеет степень, то хорошо известно, что есть корни, если учесть множественность. Чтобы понять, что подразумевается под множественностью, возьмем, например,. Считается, что этот многочлен имеет два корня, оба равны 3.

Человек изучает «теорему о факторах», обычно во втором курсе алгебры, как способ найти все корни, являющиеся рациональными числами. Также можно научиться находить корни всех квадратичных многочленов, используя при необходимости квадратные корни (возникающие из дискриминанта).Существуют более сложные формулы для выражения корней многочленов кубической и четвертой степени, а также ряд численных методов аппроксимации корней произвольных многочленов. В них используются методы комплексного анализа, а также сложные численные алгоритмы, и это действительно область постоянных исследований и разработок.

Системы линейных уравнений часто решаются с использованием метода исключения Гаусса или связанных методов. Это также обычно встречается в программах средней школы или колледжа по математике.Для нахождения корней одновременных систем нелинейных уравнений необходимы более совершенные методы. Аналогичные замечания относятся к работе с системами неравенств: линейный случай может быть обработан с использованием методов, описанных в курсах линейной алгебры, тогда как полиномиальные системы более высокой степени обычно требуют более сложных вычислительных инструментов.

Как Wolfram | Alpha решает уравнения

Для решения уравнений Wolfram | Alpha вызывает функции Solve и Reduce языка Wolfram Language, которые содержат широкий спектр методов для всех видов алгебры, от основных линейных и квадратных уравнений до многомерных нелинейных систем.В некоторых случаях используются методы линейной алгебры, такие как исключение Гаусса, с оптимизацией для повышения скорости и надежности. Другие операции полагаются на теоремы и алгоритмы из теории чисел, абстрактной алгебры и других сложных областей для вычисления результатов. Эти методы тщательно спроектированы и выбраны, чтобы позволить Wolfram | Alpha решать самые разнообразные проблемы, а также минимизировать время вычислений.

Хотя такие методы полезны для прямых решений, для системы также важно понимать, как человек решит ту же проблему.В результате в Wolfram | Alpha также есть отдельные алгоритмы для пошагового отображения алгебраических операций с использованием классических методов, которые людям легко распознать и которым легко следовать. Это включает в себя исключение, замену, квадратную формулу, правило Крамера и многое другое.

3×3 Решатель Системы Уравнений

О правиле Крамера

Этот калькулятор использует правило Крамера для решения систем трех уравнений с тремя неизвестные. Правило Крамера можно сформулировать следующим образом:

Учитывая систему:

$$ \ begin {выровнено} a_1x + b_1y + c_1z = d_1 \\ а_2x + b_2y + c_2z = d_2 \\ a_3x + b_3y + c_3z = d_3 \ end {выровнен} $$

с

$$ D = \ left | \ begin {array} {ccc} a_1 и b_1 и c_1 \\ a_2 и b_2 и c_2 \\ a_3 & b_3 & c_3 \\ \ end {array} \ right | \ ne 0 $$ $$ D_x = \ left | \ begin {array} {ccc} d_1 & b_1 & c_1 \\ d_2 & b_2 & c_2 \\ d_3 & b_3 & c_3 \\ \ end {array} \ right | $$ $$ D_y = \ left | \ begin {array} {ccc} a_1 и d_1 и c_1 \\ a_2 & d_2 & c_2 \\ a_3 & d_3 & c_3 \\ \ end {array} \ right | $$ $$ D_z = \ left | \ begin {array} {ccc} a_1 и b_1 и d_1 \\ а_2 и b_2 и d_2 \\ a_3 & b_3 & d_3 \\ \ end {array} \ right | $$

, то решение этой системы:

$$ x = \ frac {D_x} {D} $$ $$ y = \ frac {D_y} {D} $$ $$ z = \ frac {D_z} {D} $$

Пример: Решите систему уравнений, используя правило Крамера

$$ \ begin {выровнено} 4x + 5y -2z = & -14 \\ 7x — ~ y + 2z = & 42 \\ 3x + ~ y + 4z = & 28 \\ \ end {выровнен} $$

Решение: Сначала мы вычисляем $ D, ~ D_x, ~ D_y $ и $ D_z $.

$$ \ begin {выровнено} & D ~~ = \ left | \ begin {массив} {ccc} {\ color {blue} {4}} & {\ color {red} {~ 5}} & {\ color {green} {- 2}} \\ {\ color {blue} {7}} & {\ color {red} {- 1}} & {\ color {green} {~ 2}} \\ {\ color {blue} {3}} & {\ color {red} {~ 1}} & {\ color {green} {~ 4}} \ end {array} \ right | = -16 + 30-14-6-8-140 = -154 \\ & D_x = \ left | \ begin {массив} {ccc} -14 & {\ color {red} {~ 5}} & {\ color {green} {- 2}} \\ ~ 42 & {\ color {red} {- 1}} & {\ color {green} {~ 2}} \\ ~ 28 & {\ color {red} {1}} & {\ color {green} {~ 4}} \ end {array} \ right | = 56 + 280 — 84 — 56 + 28 — 840 = -616 \\ & D_y = \ left | \ begin {array} {ccc} {\ color {blue} {4}} & -14 & {\ color {green} {- 2}} \\ {\ color {blue} {7}} & ~ 42 & {\ color {green} {~ 2}} \\ {\ color {blue} {3}} & ~ 28 & {\ color {green} {~ 4}} \ end {array} \ right | = 672 — 84 — 392 + 252 — 224 + 392 = 616 \\ & D_Z = \ left | \ begin {array} {ccc} {\ color {blue} {4}} & {\ color {red} {~ 5}} & -14 \\ {\ color {blue} {7}} & {\ color {red} {- 1}} & ~ 42 \\ {\ color {blue} {3}} & {\ color {red} {~ 1}} & ~ 28 \ end {array} \ right | = -112 + 630 — 98 — 42 — 168 — 980 = -770 \\ \ end {выровнен} $$

Следовательно,

$$ \ begin {выровнено} & x = \ frac {D_x} {D} = \ frac {-616} {- 154} = 4 \\ & y = \ frac {D_y} {D} = \ frac {616} {- 154} = -4 \\ & z = \ frac {D_z} {D} = \ frac {-770} {- 154} = 5 \ end {выровнен} $$

Примечание: Вы можете проверить решение с помощью вышеуказанного калькулятора

Решающих систем с исключением Гаусса

У нас есть система трех уравнений с тремя переменными.Пусть [latex] \, x \, [/ latex] будет сумма, инвестированная под 5% годовых, пусть [latex] \, y \, [/ latex] будет суммой, инвестированной под 8%, и пусть [latex] \ , z \, [/ latex] — сумма, вложенная под 9% годовых. Таким образом,

[латекс] \ begin {массив} {l} \ text {} x + y + z = 10 000 \ hfill \\ 0,05x + 0,08y + 0,09z = 770 \ hfill \\ \ text {} 2x-z = 0 \ hfill \ end {array} [/ latex]

В качестве матрицы имеем

[латекс] \ left [\ begin {array} {rrr} \ hfill 1 & \ hfill 1 & \ hfill 1 \\ \ hfill 0,05 & \ hfill 0,08 & \ hfill 0,09 \\ \ hfill 2 & \ hfill 0 & \ hfill -1 \ end {array} \ text {} | \ text {} \ begin {array} {r} \ hfill 10,000 \\ \ hfill 770 \\ \ hfill 0 \ end {array} \ right] [/ latex]

Теперь мы выполняем исключение Гаусса, чтобы получить форму строки-эшелон.

[латекс] \ begin {array} {l} \ begin {array} {l} \ hfill \\ -0.05 {R} _ {1} + {R} _ {2} = {R} _ {2} \ в \ left [\ begin {array} {rrrrrr} \ hfill 1 & \ hfill & \ hfill 1 & \ hfill & \ hfill 1 & \ hfill \\ \ hfill 0 & \ hfill & \ hfill 0,03 & \ hfill & \ hfill 0,04 & \ hfill \\ \ hfill 2 & \ hfill & \ hfill 0 & \ hfill & \ hfill -1 & \ hfill \ end {array} | \ begin {array} {rr} \ hfill & \ hfill 10,000 \\ \ hfill & \ hfill 270 \\ \ hfill & \ hfill 0 \ end {array} \ right] \ hfill \ end {array} \ hfill \\ \, \, \, \, \, \, — 2 {R} _ {1} + {R} _ {3} = {R} _ {3} \ to \ left [\ begin {array} {rrrrrr} \ hfill 1 & \ hfill & \ hfill 1 & \ hfill & \ hfill 1 & \ hfill \\ \ hfill 0 & \ hfill & \ hfill 0.03 & \ hfill & \ hfill 0.04 & \ hfill \\ \ hfill 0 & \ hfill & \ hfill -2 & \ hfill & \ hfill -3 & \ hfill \ end {array} | \ begin {array} {rr} \ hfill & \ hfill 10,000 \\ \ hfill & \ hfill 270 \\ \ hfill & \ hfill -20,000 \ end {array} \ right] \ hfill \\ \, \, \, \, \, \, \, \, \, \, \, \, \, \, \, \, \ frac {1} {0.03} {R} _ {2} = {R} _ {2} \ to \ left [\ begin {array} {rrrrrr} \ hfill 0 & \ hfill & \ hfill 1 & \ hfill & \ hfill 1 & \ hfill \\ \ hfill 0 & \ hfill & \ hfill 1 & \ hfill & \ hfill \ frac {4} {3} & \ hfill \\ \ hfill 0 & \ hfill & \ hfill -2 & \ hfill & \ hfill -3 & \ hfill \ end {array} | \ begin {array} {rr} \ hfill & \ hfill 10,000 \\ \ hfill & \ hfill 9,000 \\ \ hfill & \ hfill -20,000 \ end {array} \ right] \ hfill \\ \, \, \, \, \, \, \, \, \, \, \, \, 2 {R} _ {2} + {R} _ { 3} = {R} _ {3} \ to \ left [\ begin {array} {rrrrrr} \ hfill 1 & \ hfill & \ hfill 1 & \ hfill & \ hfill 1 & \ hfill \\ \ hfill 0 & \ hfill & \ hfill 1 & \ hfill & \ hfill \ frac {4} {3} & \ hfill \\ \ hfill 0 & \ hfill & \ hfill 0 & \ hfill & \ hfill — \ frac {1} {3} & \ hfill \ end {array} | \ begin {array} {rr} \ hfill & \ hfill 10,000 \\ \ hfill & \ hfill 9,000 \\ \ hfill & \ hfill -2,000 \ end { массив} \ right] \ hfill \ end {array} [/ latex]

Третья строка сообщает нам [латекс] \, — \ frac {1} {3} z = -2,000; \, [/ latex], таким образом [латекс] \, z = 6,000.[/ латекс]

Вторая строка сообщает нам [latex] \, y + \ frac {4} {3} z = 9000. \, [/ Latex] Подставляя [latex] \, z = 6000, [/ latex], мы получаем

[латекс] \ begin {array} {r} \ hfill y + \ frac {4} {3} \ left (6000 \ right) = 9000 \\ \ hfill y + 8000 = 9000 \\ \ hfill y = 1000 \ end {array} [/ latex]

Первая строка сообщает нам [латекс] \, x + y + z = 10,000. \, [/ Latex] Замена [latex] \, y = 1,000 \, [/ latex] и [latex] \, z = 6,000, [/ latex] получаем

[латекс] \ begin {array} {l} x + 1 000 + 6 000 = 10 000 \ hfill \\ \ text {} x = 3 000 \ text {} \ hfill \ end {array} [/ latex]

Ответ: 3000 долларов вложены под 5%, 1000 долларов вложены под 8% и 6000 долларов вложены под 9%.

Как решить линейные системы с помощью исключения Гаусса — видео и стенограмма урока

Расширенная матрица

Помните, что матрица — это просто прямоугольный массив значений, помещенных в строки и столбцы. Сначала нам нужно превратить нашу линейную систему в матричную форму, превратив ее в расширенную матрицу. Расширенная матрица представляет собой комбинацию двух матриц. В нашем случае у нас есть матрица для коэффициентов левой части уравнения и еще одна матрица для правой части уравнения.

Напомним, что преобразование системы уравнений в матричную форму включает в себя выделение только коэффициентов вместе с их соответствующими знаками после их организации так, чтобы член x сначала сопровождался членом y , а затем членом z , знак равенства, а затем константа. Мы можем использовать вертикальную линию или несколько точек на вертикальной линии, чтобы обозначить наш знак равенства. Наша линейная система уже организована должным образом, поэтому все, что нам нужно сделать, это выделить наши коэффициенты.В нашей первой строке будет 1, 1, 1, | а затем 5. Во второй строке 2, 0, -1, | и 4. В нашей третьей строке 0, 3, 1, | и 2. Наша матрица выглядит так:

Исключение Гаусса

Теперь мы можем использовать исключение Гаусса, чтобы помочь нам решить эту линейную систему. Исключение Гаусса — это манипулирование расширенной матрицей до тех пор, пока мы не получим матрицу, которая представляет левую часть уравнений в верхней треугольной форме .Это означает, что мы хотим, чтобы все нули находились ниже главной диагонали. Эта главная диагональ начинается в верхнем левом углу и заканчивается в правом нижнем углу матрицы коэффициентов. Другими словами, мы хотим манипулировать матрицей так, чтобы 2 во второй строке и 0 и 3 в третьей строке были равны 0.

Чтобы заменить эти числа на нули, мы будем использовать наши операции со строками матрицы. Чтобы превратить наши первые 2 в 0, мы умножаем нашу первую строку на -2, а затем добавляем ее ко второй строке, чтобы создать новую вторую строку.Получаем новую вторую строку 0, -2, -3, | и -6. Теперь, чтобы заменить 3 в третьей строке на 0, мы будем использовать эту новую вторую строку в сочетании с третьей строкой. Мы умножим вторую строку на 3 и прибавим ее к третьей строке, умноженной на 2. Мы получим новую третью строку 0, 0, -7, | и -14.

Мы не переписывали первую строку, потому что нам не нужно менять это уравнение. Мы только умножили первую строку на 3, чтобы вычесть ее из второй строки.Помните, что в алгебре всякий раз, когда мы умножаем уравнение на любую константу, уравнение вообще не меняется; числа просто увеличиваются в несколько раз.

Теперь, когда у нас есть нули под главной диагональю, мы закончили с помощью исключения Гаусса. Теперь мы можем продолжить и решить нашу линейную систему.

Решение системы

Обратите внимание, насколько легко решить эту проблему. Если мы выпишем наши линейные уравнения, мы получим x + y + z = 5, -2 y -3 z = -6 и -7 z = -14.Мы можем немедленно решить третье уравнение для z , чтобы получить z = -14 / -7 = 2. Затем мы можем заменить это значение на z во второе уравнение, чтобы найти следующую переменную, y . Получаем -2 y -3 (2) = -6. Получается -2 y -6 = -6. Чтобы решить относительно y , мы прибавляем 6 к обеим сторонам и получаем -2 y = 0. Разделив обе стороны на -2, мы получаем y = 0. Итак, теперь у нас есть y = 0 и z = 2.Чтобы решить нашу последнюю переменную, x , мы можем использовать наше самое первое уравнение. Подставляя эти два значения, мы получаем x + 0 + 2 = 5. Решая это для x , мы получаем x = 3. Итак, наш окончательный ответ: x = 3, y = 0, и z = 2. Мы также можем записать это в точечной форме, например: (3, 0, 2).

Резюме урока

Давайте рассмотрим то, что мы узнали. Мы узнали, что линейных систем представляют собой наборы линейных уравнений.Линейная система имеет такое же количество уравнений, как и переменных, поскольку нам нужно по одному уравнению для каждой переменной, чтобы решить такую ​​систему уравнений. В методе, о котором мы говорили в этом уроке, используется метод с исключением Гаусса , метод решения системы уравнений, который включает в себя манипулирование матрицей так, чтобы все элементы ниже главной диагонали равнялись нулю.

Добавить комментарий

Ваш адрес email не будет опубликован. Обязательные поля помечены *